Nurses QP Previous

You might also like

Download as docx, pdf, or txt
Download as docx, pdf, or txt
You are on page 1of 62

Windshield survey is done when?

during scene size-up


Signs and symptoms of a tension pneumothorax include:
- AMS
- signs of shock (P/C/D, hypotension)
- JVD
- diminished/absent lung sounds on affected side
What priority is increased bag resistance in trauma?
the lowest
What sort of assessment is done for an elderly patient involved in a moderate auto
accident?
a rapid trauma assessment
Which is considered a "load-and-go" situation: open femur fracture or bilateral femur
fractures?
bilateral femur fractures
A tension pneumothorax is considered what kind of shock?
mechanical
All trauma patients EXCEPT which are fluid resuscitated until a systolic BP of 80-90:
head injury patients
Pleural decompression entails what?
- mid-clavicular
- second intercostal space
- above the third rib (*do not go under the second rib to avoid damage to the blood
vessels and nerves)
If you have a pediatric patient who is grunting, this is indicative of what? How to treat?
respiratory failure; MUST provide vent. support (BVM)
If you're presented wit ha patient who comes out of a house fire with no obvious burns,
who is complaining of being light-headed and dizzy, suspect what?
toxic inhalation exposure
Knee dislocations are most likely to be associated with what other damage?
damage to nerves and arteries
If you are presented with a trauma patient who has a BP above 90, what rate is the IV to
flow?
"KVO"
A pregnant trauma patient with hypovolemia is treat how?
place her on her left lateral side in order to get the fetus off the vena cava
A patient with spinal precautions begins to vomit; how to treat?
1. tilt backboard
2. suction
If you are presented with a head injury patient who is showing signs and symptoms of
cerebral herniation, how to treat?
1. maintain them at a CO2 level of 30-35 and,
2. O2 sat of 95% or greater
A brief neurological exam consists of what?
1. pupils
2. GCS
3. BGL
If you have a patient who is on capnography that suddenly drops, what is going on?
circulatory collapse
If you cannot control bleeding with direct pressure, ITLS says what?
use a hemostatic agent along WITH continuous direct pressure
Difference(s) between a pneumothorax and a tension pneumothorax:
signs of shock
Emergency rescue is done in which environments?
toxic environments
How to manage cerebral herniation?
hyperventilate w/ BVM at 20 bpm
Signs and Symptoms of cerebral herniation:
1. hypertension
2. bradycardia
3. decr. LOC that rapidly progresses to coma
4. decerebrate posturing
5. dilated pupils
6. paralysis on the side opposite to the injury
Management of ventilation in a patient with a flail segment:
1. BVM at 12 bpm
2. bulky dressing
A patient who does not have a head injury, and is intubated, manage CO2 levels
where?
CO2 at 35-45
In neurogenic shock, skin?
P/W/D
What guarantees successful endotracheal intubation?
1. CO2 wave form
2. a # on wave
Physical confirmation of correct endotracheal tube intubation:
visualizing the cords
What is used to assist in visualizing the cords during intubation?
"Sellick" maneuver: posterior pressure on the cricoid cartilage to prevent gastric inflation
and vomiting
(Ex: external laryngeal manipulation is used to improve glottis visualization)
As per ITLS, NOT ALL patients need to be placed on backboards (example?)
patient with a stab wound to axillary chest
During primary assessment (ABC), what can a medic delegate a crew member to do?
1. oxygenate patient
2. hold c-spine
If you have a trauma patient who presents with signs of shock but you CANNOT find
any sign of significant trauma, what now?
MUST consider a medical problem that turned into a trauma (*diabetic)
Persistent tachycardia is an early indicator of what?
shock in an injured child
What constitutes a "load-and-go" situation?
significant mechanisms of injury (MOI)
How to know how much volume to give a trauma patient without a head injury?
20ml/kg

(*PT's wt. in lbs., add a zero. Example: Pt weighs 130lbs.; give 1300cc's of fluid, in (2)
large bore IVs
Medication to consider when dealing with crush injuries is what?
sodium bicarbonate (to reduce the acid build-up, and treat with fluids)
Fore head injury patients, the "ideal" BP is what:
systolic BP of 110-120
Signs and symptoms of cardiac tamponade include:
- hypotension
- JVD
- muffled heart tones
- pulses paradoxical
Airway management will interrupt what?
the only thing that would interrupt a primary assessment
Isolated sternum fractures should be delt with how?
- by placing the patient on the cardiac monitor
If a patient presents with stridor from trauma, what should be done?
intubate IMMEDIATELY
What is a Scene Size-Up?
Observations made and actions taken at a trauma scene before actually approaching
the patient. It is the initial step in the ITLS Primary Survey
What does "OPIM" stand for?
Other Potentially Infectious Material
What equipment should you always take with you for trauma patients?
-PPE (Personal Protection Equipment)
-Patient transport device (stretcher, long spine board, and so on) with effective
strapping and head motion-restriction device
-Rigid cervical extrication collar of an appropriate size
-Oxygen and airway equipment, which should include suction equipment and a BVM
(Bag-Valve Mask)
-Trauma box (bandage material, hemostatic agent, tourniquet, blood pressure cuff,
stethoscope)
What are some of the potential injury patterns for a mechanism of Injury of frontal
impact? (Name 8)
-Cervical-spine fracture
-Flail chest
-Myocardial contusion
-Pneumothorax
-Aortic disruption
-Spleen or liver laceration
-Posterior hip dislocation
-Knee dislocation
What are some of the potential injury patterns for a mechanism of lateral impact (T-
bone)? (Name 8)
-Contralateral neck sprain
-Cervical-spine fracture
-Lateral flail chest
-Pneumothorax
-Aortic disruption
-Diaphragmatic rupture
-Laceration of liver, kidney or spleen
-Pelvic fracture
What are potential injury patterns for a mechanism of ejection?
Exposure to all mechanisms (and mortality increased)
What are some of the potential injury patterns for a mechanism of Injury of pedestrian
vs. car? (Name 4)
-Head injury
-Aortic disruption
-Abdominal visceral injuries
-Fracture pelvis and lower extremities
What is "index of suspicion"?
The medical provider's estimate of a disease or injury being present in a patient. A high
index of suspicion means there is a high probability the injury is present. A low index of
suspicion means there is a low risk of the injury.
List the two basic mechanisms of motion injury
Blunt and penetrating
Identify the three collisions associated with a motor-vehicle collision (MVC)
1. Machine collision
2. Body collision
3. Organ collision resulting in rupture, shearing, or bruising
Name the five common forms of MVCs
1. Frontal-impact (head-on collision)
2. Lateral-impact (T-bone collision)
3. Rear-impact collision
4. Rollover collision
5. Rotational collision
Using the three collisions associated with a MVC, relate at least three frontal-impact
collisions to potential patient injuries to deformity of the vehicle, interior structures and
body structures.
Machine collision = Deformed front end
Body collision = Spider-web pattern of windshield
Organ collision = Coup/contracoup brain, soft-tissue injury (scalp, face, neck),
hyperextension/flexion of the cervical spine

Machine collision = Deformed front end


Body collision = Steering wheel ring fracture, deformity and column displacement
Organ collision = Traumatic tattooing of patient's skin

Machine collision = Deformity of vehicle


Body collision = Dashboard fracture and deformity
Organ collision = Facial trauma, coup/contracoup brain, hyperextension/flexion of the
cervical spine, pelvis, hip and knee trauma
Using the three collisions associated with a MVC, relate at least three lateral-impact
collisions to potential patient injuries to deformity of the vehicle, interior structures and
body structures.
Machine collision = Deformed driver or passenger side
Body collision = Degree of door deformity (ex: armrest bent, outward or inward bowing
of door)
Organ collision = This cannot be predicted by external exam alone. Instead, consider
organs beneath areas of external injury:
-Head = Coup/contracoup
-Neck = Ranging from cervical-muscle strain to fracture or subluxation with neurologic
deficit
-Upper arm and shoulder = Injuries appear on the side of the impact and are common,
as are injuries to the lower extremities
-Thorax/abdomen = Injuries from soft-tissue injuries to flail chest, lung contusion,
pneumothorax, hemothorax, or possible traumatic aortic dissection. Injuries include
those to solid and hollow organs
-Pelvis/legs = Pelvic, hip or femur fractures. Pelvic injuries may also include dislocation,
bladder rupture and urethral injuries
Describe potential injuries associated with proper and improper use of seat restraints,
headrest and air bags in a head-on collision
Proper use of seat restraint = Facial, head or neck injuries such as fractures,
dislocations or spinal-cord injuries; Clavicle fractures (at the point where the chest strap
crosses) and chest-wall injuries; Internal organ damage

Improper use of seat restraint = Abdominal or lumbar spine injury; No restraint could
possibly lead to ejection of vehicle

Improper use of headrest = (second impact) Hyperextension of the cervical spine

Proper use of air bags = Injuries from a second impact after deflation can lead to striking
the steering wheel leading to internal injuries (thus, check under the deflated air bag for
mechanical deformity; Leg, pelvis or abdominal injuries; Abrasions from the nylon bag,
corneal abrasions and superficial burns on arms in the vicinity of the airbag vents
Describe at least two potential injuries from rear-end collisions
-Hyperextension of the cervical spine (if headrest too low)
-lumbar-spine injury (if the seat breaks and falls backwards)

*Rapid forward deceleration can occur from striking something in front after being struck
from behind, thus leading to front impact injuries.
How many more times is a person likely to die if they are ejected from a vehicle during
an accident?
25
What four questions are used as a checklist in scene stabilization for a tractor accident?
1. Is the engine off?
2. Are the rear wheels locked?
3. Has the fuel situation and fire hazards been addressed?
4. Are there hydraulic fluid leaks or radiator leaks?
Describe the three assessment criteria for falls
1. Distance of fall
2. Anatomic area impacted
3. Surface struck
What are at least six anticipated injuries from a fall?
-Fractures of the feet or legs
-Hip and/or pelvic injuries
-Axial loading to the lumbar and cervical spine
-Vertebrae compression fracture
-Vertical deceleration forces to the organs
-Colles fractures of the wrists
Identify the two most common forms of penetrating injuries
Knife and gun
Discuss associated mechanisms of the two most common forms of penetrating injuries
and extent of these injuries
Knife depends on the anatomic area penetrated, length of the blade, and angle of
penetration. Low-energy injury and tissue damage confined to the direct path of the
blade.

Guns depends on the anatomic area penetrated, on type of weapon, caliber (size of
bullet), and distance from which the weapon was fired. High-energy injury and tissue
damage usually not confined to the direct path of the bullet.
What are the four injury mechanisms involved in blast injuries
1. Primary = Air blast
2. Secondary = Material (shrapnel) propelled by the blast force
3. Tertiary = Body impact with ground or object
4. Quinary = (Delayed type of injury) Hyperinflammatory state from exposure to
contaminants (ex: burns, chemical, biological, radiological)
Relate how the four injury mechanisms involved in blast injuries relate to patient
assessment
1. Primary = Air containing organs injuries (ex: Ears, lungs, gastrointestinal tract) can
lead to ruptured tympanic membranes, pneumothorax, parenchymal hemorrhage,
aveolar rupture

2. Secondary = May be penetrating or blunt, as well at higher velocities than high


powered rifles

3. Tertiary = injuries similar to being ejected from a vehicle or fallen from a height.
Injuries depend on what the person impacts

4. Quinary = Dirty bombs (dispersing toxic chemicals, biological agents or radiological


agents)
What four things can interrupt the patient assessment sequence in the ITLS Primary
Survey?
-The scene becomes unsafe
-You must treat exsanguinating hemorrhage
-You must treat an airway obstruction
-You must treat cardiac arrest

*Respiratory arrest, dyspnea, or bleeding management should be delegated to other


team members while you continue assessment of the patient.
In a team setting, when a critical condition has been found on a patient, when and how
should it be managed?
Instruct the other members of the emergency medical team to carry out the intervention
immediately
At least which 9 factors classify as a "Load N Go" patient?
1. Initial assessment reveals:
-Altered mental status
-Abnormal breathing
-Abnormal circulation (shock or uncontrolled bleeding)
2. Signs discovered during the rapid trauma survey of conditions that can rapidly lead to
shock:
-Penetrating wounds of the torso
-Abnormal chest exam (flail chest, open wound, tension pneumothorax, hemothorax)
-Tender, distended abdomen
-Pelvic instability
-Bilateral femur fractures
3. Significant mechanism of injury and/or poor general health of patient
What does "MCI" stand for?
"Multiple Casualty Incident"
What does "SMR" stand for?
"Spinal Motion Restriction"
What does "ETCO2" stand for?
"End-Tidal CO2"
What is normal ETCO2 range?
35-40 mmHg
Name at least three events that trigger the need to perform an ITLS Ongoing Exam?
-Each time the patient is moved
-Each time an intervention is performed
-Any time the patient's condition worsens
How often should the ITLS Ongoing exam be performed? (two different scenarios)
1. Every 5 minutes for unstable patients
2. Every 15 minutes for stable patients
How quickly should you identify which patients require "Load and Go"?
Within two minutes
Describe the anatomy of the respiratory system
Nose, Uvula ->
Mouth, teeth, tongue ->
->Tonsil, Epiglottis, Hyoid bone, Vocal cords, Trachea
Describe the physiology of the respiratory system starting from the nose and mouth until
gas exchange happens
The airway begins at the tip of the nose and the lips and proceeds then if through the
nose down the nasopharynx, if through the mouth, oropharynx, into the hypopharynx
and then down the larynx through the trachea and bronchi ending at the alveolocapillary
membrane.
What does "ELM" stand for?
"External Laryngeal Manipulation"
What are the major anatomic landmarks from the teeth (for placing an endotracheal
tube at the correct level for an average adult)?
15 cm -> Teeth to vocal cords
20 cm -> Teeth to sternal notch
25 cm -> Teeth to carina
How do you calculate the remaining minutes of oxygen for the three class of oxygen
tanks?
Cylinder pressure remaining (PSI) -
Safe residual pressure (200PSI) X
Constant (D=0.16, E=0.28, M=1.37) /
L/min =
Minutes remaining (flow rate)
What is the Glasgow Coma Score for an adult?
--------------
EYES
--------------
4 Open
3 To voice
2 To pain
1 No response
--------------
VERBAL
--------------
5 Oriented & alert
4 Disoriented
3 Nonsensical-speech
2 Moans, unintelligible
1 No response
--------------
MOTOR
--------------
6 Follows commands
5 Localizes pain
4 Withdraws to pain
3 Decorticate flexion
2 Decerebrate extension
1 No response
Explain the importance of observation as it relates to airway control
Continual observation of the patient to anticipate problems is essential to ensure airway
control and adequate ventilation.
What does "BIAD" stand for?
"Blind Insertion Airway Device"
What does "RSI" stand for?
"Rapid Sequence Intubation" (also referred to as "DAI", Drug-Assisted Intubation)
Describe methods to deliver supplemental oxygen to the trauma patient
1. Nasal cannula
2. Simple face mask
3. Nonrebreathing masks
4. Bag-valve-mask
By how much can you increase the oxygen percentage delivered during mouth-to-mask
breathing by placing a nasal cannula on yourself?
From 17% to around 30%
What does "FROPVD" stand for? What is it?
Flow-Restricted Oxygen-Powered Ventilation Device. Artificial ventilation device that
provides 100% oxygen at a flow rate of 40 L/min at a maximum pressure of 50 5 cm
water
What is "minute volume"? What range is usually seen?
The volume of air breathed in and out in one minute. This varies from 5 to 12 liters per
minute
What do you call it when the movement of air into and out of the lungs is unable to
maintain the carbon dioxide level below 45 mmHg
Hypoventilation
Normal ventilation by healthy lungs will produce an oxygen level of about _______ in
the blood.
100 mmHg
What does "IPPV" stand for? What is it?
"Intermittent Positive Pressure Ventilation". Actively forcing air or oxygen in through the
glottic opening (ex: using a BVM)
Briefly describe the indications, contraindications, advantages, and disadvantages of
the bag-valve mask
Indications = hypoventilating or insufficient movement of air
Contraindications = [none found in book]
Advantages = Assists in ventilation
Disadvantages = Mask air leakage. Too much force of ventilation could cause gastric
inssuflation, too quick ventilation could hyperventilate.
What is lung compliance?
The "give" or elasticity of the lungs
Briefly describe the indications, contraindications, advantages, and disadvantages of
the flow-restricted oxygen-powered ventilation devices
Indications = Medical Director's advice and patient requires 100% oxygen
Contraindications = Can't intubate patient
Advantages = Can deliver 100% oxygen, frees up your hands to address other patient's
issues
Disadvantages = Can cause or worsen pneumothorax
Predictors of difficult mask ventilation can be remembered using "BOOTS", what does it
stand for?
Beards
Obesity
Older patients
Toothlessness
Snores or Stridors
What is the 6 step process in responding to the difficulty of bag-valve-mask ventilation?
1. Reposition
2. OPA/NPA
3. Two-person bag
4. Consider obstruction
5. BIAD
6. Laryngoscopy and intubation
Suction units should always be stored with which other essential tool?
Oxygen kit
Briefly describe the indications, contraindications, advantages, and disadvantages of
the blind insertion airway devices
Indications = Can't get an OPA/NPA inserted
Contraindications = [None found in book]
Advantages = Can insert without having to visualize the larynx, easier and faster to
insert than an ET tube
Disadvantages = Not as effective as ET tubes in preventing aspiration
Briefly describe the indications, contraindications, advantages, and disadvantages of
the endotracheal intubation
Indications =Jaw thrust lift alone insufficient to keep tongue from blocking airway
Contraindications = Gag reflex, SMR making it difficult
Advantages = opens airway where tongue or other restrictions made it difficult
Disadvantages = Gag reflex, improper insertion could lead air to stomach
Briefly describe the indications, contraindications, advantages, and disadvantages of
the nasopharyngeal airways
Indications = tongue or epiglottis fell against posterior pharyngeal wall
Contraindications = [none found in book]
Advantages = Better tolerated than an OPA with an intact gag reflex
Disadvantages = bleeding and trauma to the nasal mucosa is common, mild
hemorrhage from the nose after insertion
What length of an ET tube can be cut to serve as an NPA?
6 or 6.6 mm
Briefly describe the indications, contraindications, advantages, and disadvantages of
the oropharyngeal airways
Indications = tongue fell against posterior pharyngeal wall
Contraindications = Gag reflex
Advantages = maintains open airway
Disadvantages = Gag reflex, improper insertion could lead air to stomach
Describe the predictors of difficult mask ventilation and endotracheal intubation
Beards
Obesity
Older patients
Toothlessness
Snores or Stridor
Describe apneic oxygenation
Most commonly provided using nasal cannulae in addition to a face mask
Describe external laryngeal manipulation
Manipulation of the thyroid cartilage can help bring the vocal cords into view during ET
intubation. This is done by usually pressing the thyroid cartilage backward against the
esophagus and then upward and slightly to the patient's right side. Also known as
BURP (Back-Up-Right-Pressure)
Describe the essential components of an airway kit
[P.83, doubt this is on the test. Will confirm after I've taken it]
Never suction an adult patient for longer than _______.
15 seconds
The NPA is made to go into which nostril side?
Right
What part of the patient's body can use you as a rough guide to know which width of
NPA size you could use?
Smallest finger
How is the NPA measured?
From the nare to the tip of the ear lobe
How is the OPA measured?
From the corner of the mouth to the lower part of the external ear or the angle of the jaw
When ventilating an adult mouth-to-mask, what is the volume formula, ventilation rate
and inspiratory phase?
8-10 mL/Kg at 8-10 breaths per minute lasting 1.5 to 2 seconds each breath
An ETCO2 less than 25 mmHg can lead to _______ causing oxygen to bind tightly to
hemoglobin and not releasing it for use. This leads to tissue hypoxia with a falsely high
(even 100%) pulse oximeter reading.
alkalosis
An ETCO2 more than 50 mmHg can lead to _______ causing oxygen to bind loosely
and reduces the amount carried to the cells. This gives a low pulse oximeter reading
that does not respond to O2 therapy.
acidosis
What are 10 conditions to which make a pulse oximeter reading unreliable?
1. Poor peripheral perfusion
2. Hyperventilation
3. Hypoventilation
4. Severe anemia or exsanguinating hemorrhage
5. Hypothermia
6. Excessive patient movement
7. High ambient light
8. Nail polish or dirty fingernail
9. Carbon monoxide poisoning
10. Cynanide poisining
What does "SGA" stand for?
"Supraglottic Airways" (Also known as BIADs)
What does "MAAM" stand for?
"Medically Assisted Airway Management"
What is the pleural space?
The potential space between the visceral and parietal pleura within the thorax. In
disease or injury states, this space can fill with air, fluid, or blood.
What is the mediastinum?
The anatomic region within the thorax, located between the lungs, that contains the
heart and great vessels, trachea, major bronchi, and the esophagus.
Which cervical levels provide the function of ventilation?
C3 to C5
Identify the two major symptoms of thoracic trauma
Shortness of breath and chest pain
Describe the 10 signs of thoracic trauma
1. Chest wall contusion
2. Open wounds
3. Subcutaneous emphysema
4. Hemotypsis (coughing of blood)
5. Distended neck veins
6. Tracheal deviation
7. Asymmetrical chest movement including paradoxical motion
8. Cyanosis
9. Shock
10. TIC
List the immediate life-threatening thoracic injuries (Deadly dozen)
(Found in ITLS Primary Survey)
1. Airway obstruction
2. Flail chest
3. Open pneumothorax
4. Massive hemothorax
5. Tension pneumothorax
6. Cardiac tamponade
(Found in ITLS Secondary Survey/Hospital Evaluation)
7. Myocardial contusion
8. Traumatic aortic rupture
9. Tracheal or bronchial tree injury
10. Diaphragmatic tears
11. Pulmonary contusion
12. Blast injuries
What is flail chest?
The fracture of two or more adjacent ribs in two or more places, causing instability of the
chest wall and paradoxical movement of the "flail segment" in a spontaneously
breathing patient.
Describe flail chest airway management for small and large flails
Small: Oxygen and continous positive airway pressure (CPAP) ventilation may be
sufficient.

Large: ET intubation and assisted ventilation with positive end-expiratory pressure


(PEEP).
What is simple pneumothorax?
The presence of air in the pleural space that causes the lung to separate from the chest
wall and can compromise the mechanics of breathing.
Explain the pathophysiology of an open pneumothorax
Result of accumulation of air in the potential space between the visceral and parietal
pleura secondary to injury (> 3cm in diameter). This results in at least a partial collapse
of the lung.
Explain the nine steps in the management of an open pneumothorax
1. Ensure an open airway
2. Administer high-flow oxygen. Assist ventilation if necessary
3. Initially, seal the wound with your gloved hand. Then place a chest seal (with an exit)
over the defect
4. Load and go
5. Insert a large-bore IV
6. Monitor the heart, and note heart tones for comparison later
7. Monitor oxygen saturation with a pulse oximeter and end-tidal CO2
8. Transport rapidly to appropriate hospital
9. Notify medical direction early
Explain the hypovolemic and respiratory compromise pathophysiology of massive
hemothorax
When at least 1.5L (of the 3L) of blood loss into the plural space within the thoracic
cavity. As blood accumulates within the pleural space, the lung on the affected side is
compressed.
Patient may be hypotensive from blood loss and compression of the heart or great
veins. Anxiety and confusion are produced by hypovolemia and hypoxemia. Clinical
signs of shock may be apparent. The neck veins are usually flat (distended if
mediastinal compression), decreased breath sounds and dullness to percussion on
affected side.
Explain the 6 step management process of massive hemothorax
1. Ensure an open airway
2. Apply high-flow oxygen.
3. Load and go
4. Notify medical direction early
5. Treat for shock. Keep blood pressure just high enough to maintain perfusion (80-90
mmHg systolic)
6. Observe for the possible development of tension hemopneumothorax
Describe eight clinical signs of a tension pneumothorax in conjunction with appropriate
management.
1. Dyspnea (laboured breathing)
2. Anxiety
3. Tachypnea
4. Distended neck veins
5. Possible tracheal deviation away from affected side
6. Diminished breath sounds on affected side
7. Hyperresonance when percussed
8. Shock with hypotension will follow
List three indications to perform emergency chest decompression
1. Respiratory distress and cyanosis
2. Loss of the radial pulse (late shock)
3. Decreasing level of consciousness.
Describe the appropriate six management steps of a tension pneumothorax
1. Ensure an open airway
2. Administer high-flow oxygen.
3. Decompress the affected side of the chest upon emergency indications
4. Load and go
5. Transport rapidly to appropriate hospital
6. Notify medical direction early
Contrast the difference between tension pneumothorax with the clinical signs of a
massive hemothorax.
Skin= TP: Cyanotic; MH: Pale/ashen
Neck= TP: Veins distended; MH: Flat
Trachea= TP: Deviated Trachea; MH: not deviated
Percussion note= TP: Hyperresonant; MH: Dull
What is paradoxical pulse? (pulsus paradoxus)
Where the radial pulse is not felt with inspiration.
Identify the physical findings (including Beck's triad) of cardiac tamponade
Hypotension with narrow pulse pressure and Beck's triad (distended neck veins, muffled
heart sounds, and hypotension)
What does "BCI" stand for?
"Blunt Cardiac Injury"
What are the two most common blunt cardiac injuries
Contusion of the right atrium and right ventricle
Explain the 10 steps of the management associated with blunt injury to the chest
1. Ensure open airway
2. Administer high-flow oxygen
3. Load and go
4. Transport rapidly to appropriate hospital
5. Notify medical direction early
6. Apply cardiac monitor
7. If available, perform a 12-lead ECG (including V4R)
8. Treat for shock. Keep blood pressure just high enough to maintain perfusion (80-90
mmHg systolic)
9. Treat dysrhythmias as they present (based on AHA/ILCOR guidelines)
10. Watch for other complications, including hemothorax and pneomothorax.
Explain the advantages and disadvantages of needle decompression of a tension
pneumothorax by the lateral approach
Advantages:
-Chest wall is thinner here so shorter needle will more likely work
-Less likely to inadvertently cause hemorrhage from vascular structures
-Can be done quickly without having to remove body armour.

Disadvantages & Complications:


-More likely be dislodged if moving the patient.
-Can be more difficult to reach this area when the patient is in the ambulance
(especially on the right side)
-Laceration of the intercostal vessels may cause hemorrhage
-Placing the needle to low can lacerate the liver or spleen, and inserting the needle to
high can can lacerate the axillary artery, vein, or network of nerves known as the
brachial plexus
-Laceration of the lung is possible
-Increased risk of catheter kinking
-Risk of infection
Explain the advantages, disadvantages, and complications of needle decompression of
a tension pneumothorax by the anterior approach
Advantages:
-Air tends to accumulate in the anterior in a supine patient
-Easier to access when the patient is on the stretcher.
-Monitoring of the site is easier, especially less chance of moving the needle when
moving the patient.

Disadvantages & Complications:


-Proper needle length (6-9 cm) must be used.
-Laceration of the intercostal vessels may cause hemorrhage.
-Creation of a pneumothorax can occur if diagnosed wrong.
-Laceration of the lung is possible.
-Risk of infection.
Describe the three indications for emergency decompression of a tension
pneumothorax
1. Respiratory distress and cyanosis
2. Signs of shock
3. Decreasing level of conciousness
What is neurogenic (Spine) shock?
Shock caused by spinal connections to the adrenal glands and to the blood vessels are
interrupted and the vasoconstrictors, epinepherine and norepinephrine, are not
produced. Without the vasoconstrictors the blood vessels dilate and redistribute blood
flow to a larger vascular volume causing a relative hypovolemia and the heart muscle
cannot be stimulated to contract harder and faster.
What is mechanical shock?
Traumatic or medical condition that slows or prevents the venous return of blood can
cause shock by lowering cardiac output and thus oxygen delivery to the tissues.
Likewise, anything that obstructs the flow of blood to or thorugh the heart can cause
shock. These include: tension pneumothorax and cardiac tamponade.
What is cardiogenic shock?
When the heart muscle itself is damaged. Pumping strength is reduced, cardiac output
falls, and blood pressure goes down. These include: myocardial contusion or
myocardial infarction.
List the four components of the vascular system necessary for normal tissue perfusion
-Adequate air exchange in the lungs to allow oxygen to enter the blood: oxygenation.
-Intact vascular system to deliver oxygenated blood throughout the body: the blood
vessels.
-Adequate volume of fluid in the vascular system: red blood cells and plasma
-Functioning pump: the heart
"Shock state is one of _______ tissue perfusion".
low
Describe the 11 symptoms and signs of shock in the order that they develop, from the
very least to the very worst
(Compensated)
1. Weakness and lightheadedness
2. Pallor
3. Tachycardia
4. Diaphoresis
5. Tachypnea
6. Decreased urinary output
7. Weakened peripheral pulses
8. Thirst
(Decompensated)
9. Hypotension
10. Altered mental status
11. Cardiac arrest
Many of the symptoms of shock of any etiology are caused by the release of _______.
catecholamines
In shock, to which three areas does the blood shunt to?
Heart, lungs and brain.
When does the patient in "compensated" shock suddenly "crashes" (decompensates)?
Prolonged tissue hypoxia leads to worsening acidosis, which ultimately causes a loss of
response to catecholamines, worsening the drop in blood pressure.
What seven factors are described as early shock symptoms?
1. Loss of approximately 15% to 25% of blood volume
2. Slight to moderate tachycardia
3. Pallor
4. Narrowed pulse pressure
5. Thirst
6. Weakness
7. Possible delayed capillary refill
What two factors are described as late shock symptoms?
1. Loss of approximately 30% to 45% of blood volume
2. Hypotension
Lowest pressure at which the radial pulse can usually be felt begins at a systolic
pressure of about ___ mmHg, the femoral pulse at ___ mmHg and the carotid pulse at
___ mmHg.
80, 70, 60
Sustained tachycardia (generally considered to be above 100 in adults) must be
suspected of having occult _______ until proven otherwise.
hemorrhage
Some patients in shock may not develop tachycardia. What are at least two reasons?
1. Neurogenic shock while being hypotensive
2. Beta-blockers and calcium channel blocking medications
Children are unable to _______ their stroke volume so their cardiac output is very
dependant on their heart rate.
increase
Falling height of a capnography waveform may be one of the first indicators that a
patient is going into a _______ state.
shock
Describe the four common clinical shock syndromes
1. Low-volume/hemorrhagic shock (absolute hypovolemia) is caused by hemorrhage or
other major body fluid loss (diarrhea, vomiting, and "third spacing" due to burns,
peritonitis [inflammation of the peritoneum], and other causes). Think of this as the tank
is no longer full.

2. Distributive/High-space/Warm/Vasodilatory shock (relative hypovolemia) is caused by


spine injury, vasovagal syncope, sepsis, and certain drug overdoses that dilate the
blood vessels (the arterioles) and redistribute blood flow to a larger vascular volume,
thus causing reduced pressure in the vascular system (low blood pressure). Think of
this as the tank has increased in size without having more fluid placed into it.

3. Mechanical/Obstructive shock is caused by an actual physical blocking of the large


blood vessels in the chest. Think of this as blood flow into the heart or lungs being
blocked.

4. Cardiogenic shock is produced by a problem with the heart itself. This is caused by a
damaged heart which can reduce the heart's ability to pump, thus dropping blood
pressure. Think of this as the pump is not working as well as it should.
Absolute hypovolemic shock can be the result of conditions other than hemorrhage
caused by trauma. Name at least two.
1. Severe diarrhea
2. Bowel obstruction
Explain the pathophysiology of hemorrhagic shock
1. When blood volume is lost, "sensors" in the major blood vessels signal the adrenal
gland and the nerves of the sympathetic nervous system to secrete catecholamines
2. They cause vasoconstriction and thus further shrink the vascular space and maintain
perfusion pressure to the brain and heart.
3. Artery then shrinks in width due to blood loss, becoming more threadlike in size
(hence, thready pulse)
4. Usually tachycardia, paleness and flat veins is noted
The average-size adult has around 5L of blood in the vascular system. How much more
can it hold if the arterioles were to fully dilate?
5 times more (25L)
What else is the sympathetic nervous system called?
Thoracolumbar autonomic nervous system
Explain the pathophysiology of distributive/neurogenic shock
1. Neurogenic shock occurs most typically after an injury to the spinal cord
2. Damage to the spinal cord in the neck area can prevent the brain from being able to
send out sympathetic nervous system signals.
3. Thus, it is prevented from raising the pulse rate, heart's force of contraction, or from
constricting the peripheral arterioles.
4. This disruption can result in the loss of the normal vascular tone and in the inability of
the body to compensate for any accompanying hemorrhage.
5. No catecholamine release means no pallor (vasoconstriction), tachycardia or
sweating.
6. There will be decreased blood pressure, but the heart rate will be normal or slow, and
the skin usually warm, dry, and pink.
7. There may be paralysis or sensory deficit corresponding to the spinal-cord injury, as
well, lack of chest movement (diaphragmatic breathing instead) when a deep breath is
taken.
8. In males, priapism (prolonged erection of the shaft of the penis) may be present.
Name at least six types of drugs in an overdose and chemical exposures that can cause
hypovolemia.
1. Nitroglycerin
2. Beta-blockers
3. Calcium channel blockers
4. Antihypertensive medications
5. Cyanide
6. Ethyl alcohol (found in legal alcoholic beverages)
In the average adult, the heart pumps out how many litres of blood per minute?
5
Explain the pathophysiology of mechanical shock
1. Traumatic or medical condition that slows or prevents the venous return of blood to
the heart.
2. Lowers cardiac output and thus oxygen delivery to the tissues.
Which ventricle usually gets the myocardial contusion? Why?
Right. Because it is more anterior than the left side, as well as it can collide with the
sternum.
Explain the pathophysiology of cardiogenic shock
1. Heart muscle itself is damaged
2. Pumping strength is reduced, cardiac output falls and blood pressure goes down.
4. Possible dysrhythmia
5. Blood backs up into the venous system, resulting in distended neck veins.
6. Lung and peripheral perfusion drops, leading to dyspnea and cyanosis.
7. Because the patient is in shock with an intact spinal cord, catecholamines are
released, developing tachycardia and diaphoresis
How quickly can a traumatic cardiac tamponade to a circulatory arrest occur?
5-10 minutes
Normal Hgb is ___ grams per 100mL. A patient has to have at least ___ grams of
deoxygenated hemoglobin (Hgb) per 100mL of blood for cyanosis to occur. Someone
bleeding to death may not have enough hemoglobin around to manifest cyanosis.
14, 5
Describe the management of hemorrhage that can be controlled
1. Apply direct pressure to the wound

*If unsuccessful, apply a tourniquet


What do you do when you encounter a hemorrhage that you cannot be controlled?
Transport rapidly to the appropriate facility.
The patient with a severe head injury (Glasgow Coma Score of 8 or less) and shock
cannot tolerate _______. Therefore, if necessary, they should be fluid resuscitated to a
blood pressure of ___ mmHg systolic to maintain a cerebral perfusion pressure of at
least ___ mmHg.
hypotension, 100 [also in book says 90-100], 60
Describe the four points of managing a nonhemorrhagic shock syndrome (mechanical,
cardiogenic and high-space)
1. High-flow oxygen
2. Rapid transport
3. supine positioning
4. IV line placement (usually en route)
Discuss the use of hemostatic agents for uncontrolled extremity hemorrhage
If you cannot stop the bleeding with pressure or a tourniquet, you may use a hemostatic
agent (apply pressure for at least 2 minutes). It is an "adjunct" to assist in controlling
hemorrhage, not a hemorrhage control by itself. It's part of an overall hemorrhage
control protocol.
What does "TBI" stand for?
"Traumatic Brain Injury"
Describe six anatomical parts of the head and brain
1. Scalp
2. Skull
3. Fibrous coverings of the brain (meninges: dura mater ["tough mother'], arachnoid
mater, pia mater ["soft mother"])
4. Brain tissue
5. Cerebrospinal fluid (found between the arachnoid and pia mater)
6. Vascular compartments
What does "CSF" stand for?
"Cerebrospinal Fluid"
What does "ICP" stand for? When is it considered dangerous?
"Intercranial Pressure", >15 mmHg
Describe the "Monro-Kellie Doctrine"
It states that the contents of the skull (brain, blood and CSF) are at a fixed volume and a
change in volume of one is compensated by a decrease in the other two.
The only significant opening through which the pressure in the brain can be released is
the _______ _______ at the base of the skull.
foramen magnum
Explain the difference between primary and secondary brain injury
Primary brain injury occurs as an immediate damage to the brain tissue that is a direct
result of an injury force, while secondary brain injury is the result of hypoxia and/or
decreased perfusion of brain tissue after primary injury.
Describe the 5 main mechanisms for the development of secondary brain injury
1. Swelling, from the primary injury, can cause a decrease in perfusion.
2. Hypoxia or hypotension may occur insulting the brain
3. Bruising or injury of the brain results in vasodilation with increased blood flow to the
injured area
4. Since there is no extra space inside the skull, the swelling increase intracerebral
pressure which leads to a decreased cerebral blood flow that causes further brain injury
5. This process may take hours to develop
What does "CPP" stand for?
"Cerebral Perfusion Pressure"
What does "MAP" stand for? What is it?
"Mean Arterial Pressure". The sum of the diastolic blood pressure plus one-third of the
pulse pressure (systolic blood pressure minus the diastolic blood pressure).
At what pressure can cerebral herniation occur?
>25 mmHg
What is "Cushing's reflex"?
A reflex in which the body responds to markedly elevated ICP by raising both the
diastolic and systolic blood pressure, reduction of heart rate, and irregular respiration.
Also called "Cushion's response", this happens late in the course of head injury.
What does "ARDS" stand for?
"Adult Respiratory Distress Syndrome"
Recognize and describe the management of the cerebral herniation syndrome
Decreased LOC (GCS<9) leading to coma, dilated pupils (and outward-downward
deviation of the eye on the side of the injury), paralysis on the opposite side of the
injury, hypertension, bradycardia or decerebrate posturing.
Management: (Hyper)ventilate every 3 seconds for an adult, every 2.5 seconds for a
child (1-17), and every 3 seconds for an infant. Keep ETCO2 between 30-35.
What is "no-reflow phenomenon"?
The inability of restoring oxygenation and blood pressure to restore perfusion to the
cortex after an anoxic episode of 4 to 6 minutes or more. Hypothermia seems to protect
against this phenomenon.
What is an acute epidural hematoma?
Usually a tear in the middle of the meningeal artery that runs along the inside of the
skull in the temporal region, often caused by a linear skull fracture.
What is an acute subdural hematoma?
Result of bleeding from bridging veins and/or from arteries complicated with brain
contusion. Bleeding from the veins will develop symptoms slower than if they were from
arteries.
What are six other causes for dilated pupils that are not due to TBI?
1. Hypothermia
2. Lightning strike
3. Anoxia (absence of oxygen)
4. Optic nerve injury
5. Drug effect
6. Direct trauma to the eye
If the patient has a normal level of consciousness, the dilated pupil is or is not from the
head injury?
is not
Describe seven steps in prehospital management of the patient with a traumatic brain
injury
1. Secure the airway, and provide 100% oxygenation
2. Use a pulse oximeter aiming for a 95% SpO2, minimum of 90%
3. Ventilate 6-8 seconds per breath (ETCO2 between 35-45 mmHg)
4. Use SMR if necessary and elevate head to reduce ICP
5. Record vital signs.
6. Repeat the ITLS Ongoing Exam every 5 minutes
7. Insert two large-bore IV catheters
Identify four potential problems in the management of the patient with a traumatic brain
injury
1. Hyperventilating (less than 6 seconds per breath)
2. If intubated, vomiting and improper placement of ET tube causes challenges
3. Agitated and combative patients can raise ICP and cervical-spine injuries
4. Sedation can complicate neurological evaluation
When should SMR be used in relationship to patient safety?
Only when there is a clinical indication, both for the comfort of the patient and to reduce
secondary injuries from it.
Describe at least four elements of injury, history and assessment that may assist in
determining which patients will benefit from SMR
1. Acute spine injuries that are unstable and have not already caused irreversible
damage.
2. Patients who have a spinal-column injuries.
3. Injuries to the spinal-column, even if they are unstable, are adequately stabilized at
the site and do not require SMR of the entire spine.
4. Spine injuries at multiple sites in high speed impacts benefit from full SMR
Name two mechanism of injuries for which SMR can cause a significant decrease in
survival
Neurologic deficit or wound to the spinal area [A sample example stated as well, stab
wound to the lateral chest]
Explain the normal anatomy and physiology of the spinal column
-Bony tube composed of 33 vertebrae (7 cervical (C-spine), 12 thoracic (T-spine), 5
lumbar (L-spine), and the remainder fused together as the posterior portion of the pelvis
(5 sacral and 4 coccygeal)).
-Supports the body in an upright position, allows the use of our extremities, and protects
the delicate spinal cord.
-The vertebrae are each separated by a fibrous disc that acts as a shock absorber.
-The alignment is maintained by muscles and by strong ligaments between the
vertebrae
Explain the normal anatomy and physiology of the spinal cord
1. Electrical conduit that serves as an extension of the brainstem.
2. Continuous down to to the level of T1 to which then separates into nerves.
3. The cord is 10-13 mm in diameter and suspended in the middle of the vertebral
foramen.
4. Soft and flexible like cotton rope and surrounded and bathed by CSF.
5. Cord passes down the vertebral canal and gives off pairs of nerve roots that exit at
each vertebral level.
6. Nerve roots carry sensory signals from the body to the spinal cord and then to the
brain, and vice versa (to muscles as well).
7. Integration center for autonomic nervous system: controls heart rate, blood vessel
tone, and blood flow to skin.
What are the six common mechanisms of blunt spine injuries?
1. Hyperextension (excessive posterior movement)
2. Hyperflexion (excessive anterior movement)
3. Compression
4. Rotation
5. Lateral stress (lateral head stress movement)
6. Distraction (stretching)
What is "paresthesia"?
Abnormal sensation; a "tingling" or "burning" sensation
Using the clinical evaluation, differentiate neurogenic shock from hemorrhagic shock
Neurogenic shock is consistent with hypotension, normal skin colour and temperature,
and inappropriately slow heart rate.

With hemorrhagic shock, decreasing blood pressure, pallor, tachycardia and


diaphoretic.
Explain the difference between Emergency Rescue and rapid extrication techniques
Emergency Rescue = Immediate (within seconds) removal without SMR
Rapid extrication = Rapid removal (within minutes) using modified SMR
Describe the four essential components of a SMR system
1. Cervical collar
2. Straps
3. Head motion-restriction device
4. Airway management kit
Describe the goal and principal of SMR
The goal of SMR is to reduce or prevent secondary injury to the spine during transport.
Describe the principals of SMR
1. Should be applied appropriately to those patients who may have sustained or are at
high risk for spine injuries and cannot be adequately assessed clinically for the
presence of such injuries.
2. Stabilization of the spine must be done so that the patient is in the anatomical
(neutral) position.
3. Straps should be placed over stable bony structures.
4. Patients placed in SMR are at risk for airway problems and should be monitored at all
times.
Explain at least five reasons when to use SMR
-High-energy mechanism of injury with drug or alcohol intoxication
-Blunt trauma and altered level of consciousness
-Focal neurological complaint
-Spinal deformity, tenderness or pain
-To patients who cannot be adequately assessed clinically for the presence of such
injuries.
Explain when helmets should and should not be removed from injured patients
Should:
-When management of airway is needed
-When there isn't shoulder padding (to keep the spine more neutral)

Shouldn't:
-When airway is good
-The patient has shoulder pads (to keep the spine more neutral)
-Faceguards can be removed, or full face helmet
Identify the basic anatomy of the abdomen
Divided in 3 regions:
1. Intrathoracic abdomen
2. Intraperitoneal abdomen (True abdomen)
3. Retroperitoneal abdomen
Explain how abdominal and chest injuries may be related
Patients with multiple lower rib fractures are notorious for having severe intra-abdominal
injuries without significant abdominal pain. The severe pain from the rib fractures
becomes a distracting injury for the less noticeable abdominal pain.
Name at least four reasons when cervical collars shouldn't be used
When there is penetrating, disfiguring wounds, possible swelling or fractured mandible.
What is evisceration?
The protruding of intestinal organs through a wound
What is peritoneum?
A thin serous membrane that lines the abdominal cavity and covers the organs of the
abdomen.
Periumbilical bruising is also knows as _______.
Cullen's sign
Relate how at least two injuries apparent on the exterior of the abdomen reflect possible
damaged underlying structures
1. Periumbilical bruising raises suspicion of retroperitoneal hemorrhage
2. Distention and tenderness can be a sign of severe intra-abdominal trauma with likely
hemorrhage
List the critical interventions for patients with abdominal injuries
1. Control bleeding first (CABC)
2. Give supplemental oxygen
3. Transport immediately
4. En route, establish two large-bore IV lines of normal saline. Maintain 80-90 mmHg
systolic.
Describe the treatment required for the patient with protruding viscera
1. Remove clothing to fully expose the wound
2. Cover it with a sterile gauze dressing soaked with normal saline
3. Cover the moistened sterile dressing with an occlusive dressing (such as plastic
wrap) to prevent evaporative drying.
Describe how to identify and stabilize a pelvic fracture, why this is important, and
contraindications
Identify: High-energy blunt trauma (such as an MVC)

Stabilize: Pelvic binder/circumferential bed sheet

Why important: To allow clot formation, to minimize further blood loss

Contraindications: Patient sustained a low-energy injury, avoid pelvic binding because


applying pressure could cause further injury.
What does "PERRL" stand for? What do the pupils papillary sizes mean?
"Pupils Equal Round Reactive to Light"
<2 pinpoint
2-3 Constricted
4-5 Normal
6-8 Dilated
>8 Blown
What is a sprain?
A sudden twist of a joint with stretching or tearing of ligaments
What is a strain?
A stretching or partial tearing of a muscle or musculotendinous unit.
Discuss the major immediate and short-term complications and treatment of
amputations
Massive hemorrhage possible; Direct pressure applied to stump, use tourniquet if
needed. Cover with damp sterile dressing and elastic wrap uniformly applying pressure
over wound. Bring amputated part if possible. Small parts, rinse, put in sealed bag then
that bag in another with water and ice.
Discuss the major immediate and short-term complications and treatment of
dislocations
Dislocations: neurovascular comprimises can lead to significant disability, even
amputation; Splint as found with adequate padding and stabilization, and (possible)
analgesic medication. If loss of distal pulse, attempt to bring body part into a more
anatomic position to restore distal pulse.
Discuss the major immediate and short-term complications and treatment of fractures
Neurovascular injuries from bone fragments, swelling pressure or hematoma; Sharp
bone can cut surrounding tissue; open wounds can contaminate; Splint fracture and
(possible) analgesic medication.
Discuss the major immediate and short-term complications and treatment of open
wounds
Massive hemorrhage possible; Direct pressure or pressure dressings, exsanguinating
bleeding usually requires a tourniquet (2 if needed). If neither works, then use
hemostatic dressing (not on abdominal or chest wounds) or use newer junctional
tourniquet; Once bleeding is controlled, cover with moist sterile dressing/bandage.
Remove large contaminations (if long transport time, irrigate wound to remove smaller
particles)
Describe the potential amount of blood loss from pelvic and femur fractures
Pelvic: More than 1L
Femur: 1-2L (each femur)
Discuss the major immediate and short-term complications and treatment of
neurovascular injuries
Loss of circulation or sensation, foreign bodies or broken bone ends may impinge
delicate structures and cause them to malfunction; Check for PMS before and after
treatment. Splint as found. If there is circulation or sensation loss, correct splinting, if
not, transport quickly.
What are the major immediate complications and treatment of sprains
Pain and swelling; Treat as if it were a fracture. Splint (and possible analgesic
medication).
What are the major immediate complications and treatment of strains
Pain and swelling; Splint for comfort, use ice for swelling.
What is the treatment for impaled objects?
Do not remove object (unless obstructing airway), secure in place with bulking padding.
Discuss the pathophysiology of compartment syndrome and which extremity injuries are
most likely to develop this complication
Increased tissue pressure in a muscle compartment results in decreased blood flow,
leading to tissue hypoxia and possible muscle, nerve, and vessel impairment, and which
can be permanent if the cells die.

Lower legs are the most likely to develop this complication.


What are the late signs and symptoms of compartment syndrome called the "Five P's"?
1. Pain
2. Pallor
3. Pulselessness
4. paresthesia (numbness or tingling)
5. Paralysis
Discuss the major immediate complications and treatment of crush injuries
To the torso, it can cause asphyxia and death. To extremities, it can cause damage to
tissue, muscles and compromise circulation. Injured tissues swell decreasing perfusion.
Tissue injury after 1 hour of circulation compromise, but can tolerate up to 4 hours
before cell death. Once circulation is restored, toxins produced during anaerobic
metabolism flood the body and affect many organs, thus "crush syndrome". Heart
pumps less effectively, acute renal failure.
Management of immediate life threats followed by fluid resuscitation.
What is DCAP-BTLS?
Deformity, Contusions, Abrasions, Penetrations, Burns, Tenderness, Lacerations,
Swelling.
What is crepitation?
The sound or feel of broken fragments of bone grinding against each other
Tourniquets should not be used longer than _______ hours.
2 [that was stated on a sample exam; no more than 4 was recommended in the book]
What is open-book pelvic fracture?
A severe pelvic fracture in which the symphysis is torn apart and the anterior pelvis is
"opened" like a book.
Discuss major mechanisms of injury, associated injuries, potential complications, and
management of injuries to the pelvis
Usually caused by MVC or falls. Identified by instability or pain. Suspect hemorrhage
and shock. Apply pelvic sling. Evaluate for spine injuries. Rapid transport (use scoop
stretcher if possible).
Discuss major mechanisms of injury, associated injuries, potential complications, and
management of injuries to the clavicle and shoulder
Can be from injury to the neck, chest, or even abdomen. Injury to the radial nerve
results in patient not able to lift the hand. Check for DCAP-BTLS. Stabilize bone end to
prevent further soft-tissue injury. Apply sling or swathe as needed. Dislocated shoulders
may require a pillow between arm and chest. Pain control.
Discuss major mechanisms of injury, associated injuries, potential complications, and
management of injuries to the femur
Associated open wounds (if so, treat as fracture). When spasms develop after, it can
cause the bone to override causing more damage. Use traction splinter (unless there's a
pelvic fracture as well).
Discuss major mechanisms of injury, associated injuries, potential complications, and
management of injuries to the hip
Fallen elderly will also have knee and pelvic pain. Do not use a traction splint.
Deslocation can occur.

Posteriorly usually when hitting the dashboard of a vehicle, and needs orthopedic
emergency so to prevent sciatic nerve injury or necrosis of the femoral head. Support in
most comfortable position with pillows and splinting to uninjured leg.

Anteriorly is rare. Cannot bring leg forward in line with body, as well, difficulty putting
patient on backboard/stretcher. If vein by hip has collapsed, clot can form distally,
producing a large pulmonary embolus as soon as the hip is reduced.
Discuss major mechanisms of injury, associated injuries, potential complications, and
management of injuries to the knee
In sitting position, can have injuries to the hip. Blood vessels and nerves often injured if
in abnormal position. Restore circulation below the knee. Transport rapidly. If there is a
loss of pulse or sensation, apply gentle traction (along the long axis of the leg) by hand.
Splint in comfortable position.
Discuss major mechanisms of injury, associated injuries, potential complications, and
management of injuries to the tibia and fibula (including ankle)
Swelling and internal hemorrhage can occur over time. Use a rigid splint, air splint, or
pillow. Dress any wounds, pad exposed bone ends under a splint, and manage patient's
pain. Elevate extremity.
Discuss major mechanisms of injury, associated injuries, potential complications, and
management of injuries to the elbow
Usually by a fall onto an outstretched arm. Danger of damage to the vessels and
nerves. Splint in comfort. Check distal pulse. Do not straighten or apply traction.
Transport promptly.
Discuss major mechanisms of injury, associated injuries, potential complications, and
management of injuries to the forearm and wrist
Usually by a fall onto an outstretched arm. Falls onto the wrist frequently injure the
elbow. Immobilize with a rigid or air splint (splint hand in position of function). Risk of
compartment syndrome.
Discuss major mechanisms of injury, associated injuries, potential complications, and
management of injuries to the hand and foot
Can produce multiple open fractures and avulsions.

Hand: Roll of gauze in hand, then wrap entire hand.

Foot: Falling foot injuries often have lumbar-spine injuries. Use pillow for foot support.
When do you use a traction splint?
When you need to provide linear forces along the bone to keep ends immobilized.
Describe the complications of using a traction splint
They cannot be used in hip fractures or if there is more than one fracture in the lower
extremity. Two people are needed to apply it. Long splints need extra care so they are
not hit while moving.
Apply compressive forces to a pelvic stabilizer at the level of the _______ , not at the
iliac wings.
greater trochanters of the femur.
Under what age do children usually need padding beneath the back and shoulders on
the board?
3 [another page in the book said up to 8 years old]
Identify the basic anatomy of the skin, specifically the epidermal and dermal layers plus
structures found within.
The outer layer which is thin and we can see is called the epidermis. Underneath it is
the dermis, a thick layer which contains sensory nerves and support structure to hair
follicles, sweat glands and oil glands.
List the basic functions of the skin
Mechanical and protective barrier between the body and outside world, sealing fluids
inside, and preventing bacteria and other microorganisms from readily entering the
body.
Also a vital sensory organ input providing input to to the brain of the environment and
serves a primary role in temperature regulation.
Describe the three types of burns as a function of burn depth
Superficial (1st degree)
Partial thickness (2nd degree)
Full thickness (3rd degree)
Estimate depth of burn based on skin appearance
1st degree: red
2nd degree: mottled red
3rd degree: Pearly white and/or charred, translucent and parchment-like
What is eschar?
When skin protein denatures and hardens, forming a firm, leather-like covering.
What is erythema?
superficial reddening of the skin, usually in patches, as a result of injury or irritation
causing dilatation of blood capillaries.
Describe the "rule of nines"
[Go to->] bit.ly/rulesof9s
For small or irregular burns, how can you estimate the size?
Using the entire palm (hand) of the patient equalling 1%
Which burn areas should be treated in the hospital? (exempting 1st degree burns)
bit.ly/burnareas
Identify which patients may require transport to a burn center.
Those with partial-thickness or full-thickness burns
Describe the initial management of thermal burns
-Remove patient from source
-Manage airway
-Cut away loose clothing and jewellery
-Irrigate with clean room temperature water for about 1-2 minutes (5-10 during the
detailed survey) so as to not induce hypothermia
-Cover with clean, dry sheets and blankets.
List situations and physical signs that indicate suggest carbon monoxide poisoning
Common smoke, exhaust fumes and home space heaters.

Typical order of appearance: Headache, disturbed judgement, irritability, dizziness,


decreased vision, fainting, convulsions, unconsciousness, apnea, then death.
How does carbon monoxide cause hypoxia?
Carbon monoxide binds 257 times stronger than oxygen to the hemoglobin, therefore
taking up the "seats" on the hemoglobin that oxygen would have taken. Since the body
cannot use carbon monoxide, it starves of oxygen.
Up to how long does it take for a patient to recover from carbon monoxide, if they are
just breathing fresh air? If given high-flow oxygen by mask? Using hyperbaric oxygen?
7 hours, 1.5-2 hours, 30 minutes.
Describe the initial treatment for carbon monoxide poisoning
Remove patient from source and give high-flow oxygen.
List situations and physical signs that indicate thermal airway or inhalation injury
Situations: When a patient is injured in a confined space, sometimes even in open
spaces. Exposure to steam.

Signs: Burns of the face (singed eyebrows or nasal hair, burns in the mouth, swollen
lips), carbonaceous (sooty) sputum, swelling in the upper airway. Hoarse voice, seal-
bark cough and stridor (if so, requires immediate intubation).
Identify complications and describe the initial management of chemical burns
Skin changes can be minimal even if injury is severe. Body can absorb chemicals
causing internal organ failure. Inhalation can cause lung tissue damage with respiratory
failure.

1. Wear proper PPE


2. Remove patient from source
3. Remove patient's clothing
4. Brush off chemical powder (if eye contamination, remove contact lenses or foreign
objects)
5. Irrigate copiously with clean water or other irrigant (away from the body)
6. Wipe or gently scrape off any agent still on, continue to irrigate.
Identify complications and describe the initial management of electric burns
Complications: Cardiac arrhythmia (cardiac dysrhythmia) and respiratory arrest.
Additional surface flame burns may result if patient's clothing is ignited. Fractures and
dislocations may be present due to the violent muscle contractions. Sometimes injuries
to falls right after electrocution. Internal injuries involve muscle and nerve damage, and
possible intravascular blood coagulation.

Management:
1. Safety first.
2. Patient must be removed from source.
4. Transport rapidly and treat cardiac issues (then treat for thermal burns).
What is rhabdomyolysis?
Disentigration (lysis) or dissolution of muscle. This releases large amounts of myoglobin
into the blood, which can precipitate in the kidneys, causing renal failure.
What injury do you think occurred to a patient you find naked (or partially unclothed)
unconscious or confused, with perforated eardrums and a fern-like or splatter burn
pattern on his body?
Lightning strike
Describe effective techniques for gaining the confidence of children and their parents
Demonstrate to the parents your competence and compassion in managing the child.
Use pediatric equipment. Explain to parents what and why you are doing. Involve
parents (those who are a positive influence for the situation) as much as possible in
helping and communicating to the child.

For children, speak and act in a way that is understandable and calming. Don't ask the
child, but give them options. When necessary, be clear of what you are doing
demonstrating there is no pain involved. Distract when necessary but don't lie.
Predict pediatric injuries based on common mechanisms of injury
Falls (head), MVC (seal-belt syndrome; liver, spleen, intestines or lumbar spine),
automobile-pedestrian or bicycle (head), crashes, burns, drownings, and child abuse
(injury pattern not consistent with the mechanism; nonaccidental trauma (NAT))
Name at least three unique points to remember when performing the lTLS
Primary/Secondary Surveys in the pediatric patient
1. Bulb syringe for suction
2. Remove loose teeth
3. Pad shoulders when laying flat.
If you must intubate a child, what should you do prior to intubation?
preoxygenate 5-7 breaths of high-flow oxygen via BVM at the normal rate for the age.
What is a quick way to choose the correct ET tube for a child?
Same diameter as the tip of the child's little finger.
A child's blood volume is about ___ to ___ mL/Kg
80, 90
What does "SCIWORA" stand for?
Refers to spine injuries that are not identified on plain x-rays or a CAT scan. "Spinal
Cord Injury WithOut RAdiography"
What is kephotic deformity?
A condition caused by narrowing of the vertebral discs and gradual collapse of the
osteoporotic thoracic vertebral bodies often seen in the elderly who present in a stooped
posture with an "S" shape to the spine.
Describe seven physiologic changes associated with pregnancy
-Blood volume increases by 40%-50%
-Heat rate increases by 10%-15%
-Cardiac output increases 20%-30%
-Blood pressure decreases 5-15 mmHg
-Hematocrit/hemoglobin decreases
-PaCO2 (partial pressure of carbon dioxide) decreases
-Gastric motility (Filling and Emptying) decreases
What is supine hypotension syndrome?
The drop in blood pressure seen in a woman who is greater than 20 weeks pregnant is
in the supine position, caused by the weight of the pregnant uterus pressing on the
inferior vena cava and decreasing the return of blood to the heart by up to 30%
What is abruptio placenta?
The separation of the placenta from the wall of the uterus
What types of injuries are most commonly associated with the pregnant trauma patient
MVC: Fetal distress, fetal death, abruptio placenta, uterine rupture

Penetrating injuries: Gunshot wounds and stabbings

Domestic violence: Face and neck

Falls: Pelvic and abruptio placenta

Burns
List the considerations for assessment and management of patients in whom substance
abuse is suspected
Follow normal ITLS survey, but pay attention to speech (slurred, rambles), respiration
(very low), any needle marks and reduced pain sensory.
Define excited delirium (EXD)
A syndrome characterized by sympathetic nervous system excitation with tachycardia,
hyperthermia, and hypertension. These patients often hallucinate and are very agitated.
They are at risk for cardiac dysrhythmias and death.
What is TCPA?
Traumatic Cardiopulmonary Arrest. A grouping of conditions defined by the common
precipitating factor of trauma as the origin for the cardiac arrest.
Identify at least seven patterns in traumatic cardiac arrest for whom you should withhold
resuscitation attempts
1. Blunt trauma with no breathing, pulse, or organized rhythm on ECG on EMA arrival at
the scene.
2. Penetrating trauma with no breathing, pulse, pupillary reflexes, spontaneous
movement, or organized ECG activity.
3. Any trauma with injuries obviously incompatible with life (ex: decapitation)
4. Any trauma with evidence of significant time lapse since pulselessness, including
dependant lividity ("black and blue"), rigor mortis, etc.
5. Cardiopulmonary arrest patients in whom the mechanism of injury does correlate with
the condition (if not, initiate standard resuscitation)
6. With medical direction: EMS-witnessed cardiopulmonary arrest and 20 minutes of
unsuccessful resuscitation and the patient remains in a "nonshockable" rhythm or PEA
(Pulseless electrical activity) at a rate less than 40 beats per minute.
7. With medical direction: when transport time to the hospital is more than 15 minutes
and the preceding condition exists
Special consideration should be given to victims of near _______, _______, and
_______ in NOT withholding or terminating resuscitation of TCPA
drowning, lightning strike, and hypothermia
What does "FBAO" stand for?
Foreign Body Airway obstruction
What are the three most common bloodborne viral illness to which emergency care
providers are likely to be exposed in the provision of patient care?
Hepatitis B (HBV), Hepatitis C (HCV), and HIV infection. Contamination is through blood
only. Vaginal secretion and semen are only a risk through sexual contact.
What is "PEP"?
Post-exposure prophylaxis: The administration of medications to prevent infection from
the agent to which the person was exposed.
What should you do to help prevent contamination of airborne and droplet -transmitted
diseases?
Stay vaccinated and use a mask, gown and goggle, but if possible, put the mask on the
patient. Disinfect surfaces after use.
What procedure should you follow as a minimum if you are accidentally exposed to an
infection?
1. Thoroughly wash exposed area.
2. Notify your DICO Designated Infection Control Officer. They will assess if post-
treatment of any kind is needed.
For a spleenic injury, where will the patient probably say the pain is?
Left posterior shoulder
For a liver injury, where will the patient probably say the pain is?
Right posterior shoulder
Pressure in the oropharynx of greater than ___ cm H2O will open the esophagus and
lead to air entering the stomach (gastric insufflation). BVM's may exceed ___ cm H2O
pressure in the airway.
25, 60 [was on a sample test]
Describe the ITLS PRIMARY SURVEY (1st Phase)
PRIMARY SURVEY
Scene Size-Up
We're donning our BSI (Body Substance Isolation); Gloves, goggles (etc)
Do I see, hear, smell or sense anything dangerous?
Are there any other patients?
I'm requesting additional personnel and resources; (ALS, electric and/or fire)
Do we need special equipment?
Are there any bystanders with any info?
What is the mechanism of injury here?
Is it generalized or focused?
Is it potentially life-threatening?

Initial Assessment
What is my general impression of the patient as I approach?
Assessing patient's AVPU (Alert, Verbal, Pain or Unresponsive status)
"My name is Art and I'm with the Firefighters here to help you. Please don't move. Can
you tell us what happened?"
(if needed) "Ensure cervical spine control"
"What's your name? Do you know where you are? Do you know what day it is?"

ABCs
("Can you open your mouth and stick out your tongue for me?") Is the airway open and
clear?
Is the patient breathing?
What is the rate and quality of respiration?
Ventilations Instructions
(Abnormal respiration, altered mental status, shock or major injury?)
("Administer oxygen via a nonrebreather at 15 L/m")
(Hypoventilating (<8 or <10 breaths per minute) or inadequate movement of air?)
("Give assisted ventilation through a BVM with oxygen at 15 L/m")
(Head injury who is unresponsive and show signs of cerebral herniation?)
("Give assisted hyperventilation through a BVM every 3 seconds with oxygen at 15
L/m")

What is the rate and quality of the pulse at the wrist or neck?
Is major external bleeding present?
What is the skin colour, condition and temperature? Are capillaries refilling quickly?

Decision
(Is this a critical situation?) "Patient is a LOAD N GO"
(Are there interventions that I must make now?)
Describe the ITLS RAPID TRAUMA SURVEY
Head and Neck
Are there obvious wounds of the head or neck?
Is there subcutaneous emphysema?
Are the neck veins distended?
Does the trachea look and feel midline or deviated?
Is there deformity or tenderness of the neck?
"Stabilize the neck with a neck collar"

Chest
Is the chest symmetrical? Is there paradoxical movement or any obvious blunt or
penetrating trauma?
Are there any open wounds?
Is there TIC (Tenderness, Instability, Crepitus) of the ribs?
Are the breath sounds present and equal?
(If unequal) is the chest hyperresonant or dull?
Are heart sounds normal or decreased?

Abdomen
Are there obvious wounds of the abdomen?
Is the abdomen soft, rigid or distended? (look at pt's face)
"Is there any tenderness as I touch here?"

Pelvis
Are there obvious wounds or deformity of the pelvis?
Is there TIC?

Upper Legs
Are there obvious wounds, swelling or deformity of the upper legs?
Is there TIC?

Lower Legs
Are there obvious wounds, swelling or deformity of the lower legs?
Is there TIC?
"Can you wiggle both your toes?"
"Can you feel me squeezing both your big toes?"
(if needed) "Get the backboard ready!"

Upper Arms
Are there obvious wounds, swelling or deformity of the upper arms?
Is there TIC?
"Can you wiggle your fingers on both hands?"
"Can you feel me squeezing both your thumbs?"

Exam of Posterior (done during transfer to the backboard)


Is there DCAP-ITLS (Deformities, Contusions, Abrasions, Penetrations, Instability,
Tenderness, Lacerations or Swelling) of the back? Any edma?

Decision
(Is this a critical situation?) "Patient is a LOAD N GO"
(Are there interventions that I must make now?) (if critical, transfer to ambulance now)

SAMPLE History
"I have a few questions for you: " (What is the SAMPLE history of the patient?)

Baseline Vital Signs


Are the vital signs abnormal? RP, BP, HR

Disability (Perform now if there's altered mental status)


Are the pupils equal and reactive?
Are there signs of cerebral herniation? (Unconscious, dilated pupil(s), hypertension,
bradycardia or posturing?)
"The Glasgow Coma Score is..."
Does the patient have a medical identification device?
Describe the ITLS DETAILED EXAM
(If critical) PATCH: Let medical Direction know early of our status

(if haven't received) SAMPLE History


"I have a few questions for you: " (What is the SAMPLE history of the patient?)

VITALS
What are the vital signs? (GCS, Resp, BP, Pulse, Skin, Temp, SP02)
LOC
What is the LOC? "Level of Consciousness"
(If altered mental status) What is the blood glucose?
Are the pupils equal? Do they respond to light?
"Can you wiggle all your fingers? Can you feel me squeezing your fingers? Can you
wiggle your toes? Can you feel me squeezing your toes?"
Do we need a pulse oximeter, cardiac, EtCO2 monitor, naloxone?
(If altered mental status) "The Glasgow Coma Score now is..."

Head
Is there DCAP-BTLS of the face or head? "B is for Burns"
Are Battle's sign or Raccoon eyes present?
Is there blood or fluid draining from the ears or nose?
Is there pallor, cyanosis or diaphoresis?

Airway
("Can you open your mouth and stick out your tongue for me?") Is the airway open and
clear?
(If there are burns of the face) Are there signs of burns in the mouth or nose?

Breathing
What is the rate and quality of respiration?

Neck
Is there DCAP-BTLS of the neck?
Are the neck veins normal, flat or distended?
Is the trachea midline or deviated?

Circulation
What is the rate and quality of the pulse at the wrist or neck?
What is the skin colour, condition and temperature? Are capillaries refilling quickly?
Is all external bleeding still controlled?

Chest
Is there DCAP-BTLS of the chest?
Are there any open wounds or paradoxical movement?
Are the breath sounds present and equal?
(If unequal) Is the chest hyperresonant or dull?
Are heart sounds normal or decreased?
(If intubated) Is the ET tube still in good position?

Abdomen
Is there DCAP-BTLS of the abdomen?
Is the abdomen soft, rigid or distended?

Pelvis (Skip)

Lower Extremities
Is there DCAP-BTLS of the lower extremities?
Is there normal PMS? (Pulse, Motor and Sensation)
Is range of motion normal? (optional) -> "X, can you bend and move your legs ok?"

Upper Extremities
Is there DCAP-BTLS of the upper extremities?
Is there normal PMS?
Is range of motion normal? (optional) -> "X, can you bend and move your arms ok?"

PATCH (update)
Describe the ITLS ONGOING EXAM
Subjective Changes
Are you feeling better or worse now?

Mental Status
What is the LOC?
What is the pupillary size? Are they equal? Do they react to light?
(If altered mental status) "The Glasgow Coma Score now is..."

Reassess ABCs
Airway
("Can you open your mouth and stick out your tongue for me?") Is the airway open and
clear?
(If there are burns of the face) Are there signs of inhalation injury?

Breathing and Circulation


What is the rate and quality of the respiration?
What is the rate and quality of the pulse at the wrist or neck?
What is the blood pressure?
What is the skin colour, condition and temperature? Are capillaries refilling quickly?

Neck
Is the trachea midline or deviated?
Are the neck veins normal, flat or distended?
Is there increased swelling of the neck?

Chest
Are breath sounds present and equal?
(If unequal), is the chest hyperresonant or dull?
Are heart sounds still normal or have they become muffled?

Abdomen (If mechanism suggests possible injury)


"Is there any tenderness as I touch here?" (look at pt's face)
Is the abdomen soft, rigid or distended?

Assessment of Identified Injuries


Have there been any changes in the condition of any of the injuries I have found?
(Injuries to any extremities?) "Checking PMS..."

Check Interventions
OPQRST
Is the ET tube still patent and in the correct position?
Is the oxygen rate correct?
Is the oxygen tubing connected?
Are the IVs running at the correct rate?
Is the open chest wound still sealed?
Is the decompression needle still working?
Are there any blood soaked dressings?
Are the splints in good position?
Is the impaled object still well stabilized?
If pregnant, is she tilted to her left side?
Is the cardiac monitor attached and working?
Is the pulse oximeter attached and working?
(If intubated) Is the CO2 monitor attached and working?

I repeat the ongoing examination every 5 minutes for critical patients or every 15
minutes if stable

PATCH (update)
Which injuries are by and large responsible for the majority of mortality from trauma in the
world?
Motion
What are the three separate events in an MVC?
Machine collision, body collision, organ collision
Five forms of MVCs?
Frontal, lateral, rear, rollover, rotational
Common injuries for frontal collisions?
Windshield, whiplash, steering wheel, compression, dashboard
Common injuries for lateral collisions?
Lateral displacement, side impact
Common injuries for rear collisions?
Whiplash, spinal, deceleration
Common injuries for rollover collisions?
Axial-loading, ejections, impact, whiplash, etc.
Common injuries for rotational collisions?
frontal impact, lateral impact injuries
Should you board someone with a penetrating injury if the injury is not near the spine?
No
What are the five different blast injuries?
Primary, secondary, tertiary, quaternary, quinary
Primary blast injuries?
initial air blast
Secondary blast injuries?
shrapnel hits
Tertiary blast injuries?
body being thrown into ground or other objects
Quaternary blast injuries?
Thermal burns or respiratory injuries from inhalation of fumes
Quinary blast injuries?
contamination by chemical, biological, or radiological material
What is the highest cause of death in young people?
Trauma
What are the three things that you're allowed to interrupt the primary survey?
Scene becomes unsafe, treatment of airway obstruction, treatment of cardiac arrest
What's the goal to complete primary survey?
less than two minutes and on scene time of 5 minutes or less
What is the "fix-it" process?
Delegating to team interventions and continuing the assessment
What's the first thing to be done in a trauma?
Control bleeding then airway
If the chest is moving but you don't feel air, is the pt breathing adequately?
No
What is the general rule for all patients with multi-system trauma?
High-flow supplemental oxygen, keeping the pulse oximeter reading around 95% rather than
100%
What should be done with bleeding that is not controlled with pressure?
Tourniquet, if a hemostatic agent and pressure fails.
When should you obtain a SAMPLE history and why?
At the same time you are performing the pt assessment (another EMT can be doing this) because
you may be obtaining info from bystanders who won't be transporting.
What are the critical injuries and conditions in which you should transport immediately?
AMS, abnormal breathing, abnormal circulation, injuries that could lead to shock, significant
mechanism of injury or general health
What procedures should not be performed before rapid transport?
non-lifesaving procedures such as splinting, bandaging, IV access, or elective endotracheal
intubation
Per ITLS, when should reassessment in the ongoing exam occur?
critical patients every 5 minutes, stable patients every 15 minutes.
During what occurrences, should the ongoing exam be performed?
each time the pt is moved, each time an intervention is performed, any time the patient's
condition worsens.
When should a secondary assessment be performed?
During transport rather than on scene, unless primary survey does not reveal a critical condition
What does TIC stand for?
Tenderness, instability, and crepitus
When should a glucose check be obtained for AMS in a trauma?
Ongoing exam
What is serum lactate a good marker for?
Tissue hypoxia, predictor of shock
What is a FAST exam and what is it good for?
Focused Assessment with Sonography in Trauma, good for assessing trauma in the abdomen
What is the chest wall comprised of?
skin, subcutaneous tissue, muscle, ribs, and the neurovascular bundle (which runs around the
lower border of the rib)
How much blood can the adult thoracic cavity contain?
Up to 3 liters on each side
What does the mediastinum include?
heart, aorta, pulmonary artery, superior and inferior vena cava, trachea, major bronchi, and
esophagus
Where is the diaphragm located?
lower six ribs and xyphoid process
What nerve innervates the diaphragm and where does it originate in the cervical spine?
Phrenic nerve, originating from C3-C5
What injuries are responsible for most deaths on scene?
Chest injuries
What life-threatening thoracic should be identified immediately during the ITLS primary survey?
Airway obstruction, flail chest, open pnuemo, massive hemothorax, tension pnuemo, cardiac
tamponade
What injuries should be identified during the ITLS secondary survey?
Myocaradial contusion, traumatic aortic rupture, tracheal or bronchial tree injury, diaphragmatic
tears, pulmonary contusion, blast injuries
Can flail segments be seen in the posterial wall?
Not typically due to the muscles
How are large flail chests best treated?
ET ventilation, and assisted ventilation with PEEP
How are small flail chests best treated?
Oxygen and CPAP ventilation
What effect does an open pneumothorax have on intrathoracic pressure?
It equalizes the intrathoracic pressure and atmospheric pressures resulting in partial or complete
lung collapse
What is a massive hemothorax?
hemothorax resulting in at least 1500 cc blood loss in the pleural space
Signs of a tension pneumothorax?
decreased LOC, rapid/shallow breathing, weak/thready/absent radials,
diaphoretic/cool/clammy/cyanotic, JVD, TD, decreased/absent breath sound, hyper-resonant
sound on one side
Signs of a Massive hemothorax?
Decreased LOC, rapid/shallow breathing, weak/thready/absent radials,
cool/clammy/diaphoretic/pale, neck veins flat, trachea midline, decreased/absent breath sounds,
dull sound
What is the sellick maneuver?
Creating pressure on the cricoid at the front of the neck to close off the esophagus, pressure as
high as 100cm H20
What is the Sellick maneuver used for?
Reduce the risk of gastric regurgitation, prevent air going into the stomach
What should you do if there is trauma to the neck and you must perform the Sellick maneuver?
carefully support and stabilize the neck while performing.
What is noisy breathing?
obstructed breathing
What can combativeness be a sign of?
Hypoxia in the patient (possibly due to ET tube displacement)
Should you remove an NPA that results in mild hemorrhage and why or why not?
No, as it may disturb a clot or reactivate bleeding
What is RSI?
Rapid sequence intubation, using paralytics to intubate pations
Should you RSI someone who has spontaneous yet inadequate respiratory effort?
No, using a paralytic may result in further decrease in respiratory effort if tube is unable to be
placed successfully.
What is MMAP?
Mallampati (levels of inability to view structures, measurement 3-3-1 (3 fingers under neck, 3
fingers in mouth and one centimeter between teeth), atlanto-occipital extension (sniffing
position), and pathology (airway obstruction indicated per stridor)
What are the different levels of the mallampati score?
1- entire tonsil or tonsilor bed, 2 - upper half of tonsil, 3 - soft and hard palare, 4 - only hard
palate
Simple face mask flow rate and oxygen saturation?
10-12lpm at 40-50% O2
NRB face mask flow rate and oxygen saturation?
12-15lpm and 60-90% O2
NC flow rate and oxygen saturation?
2-6lpm and 24-30% O2
What flow rate should supplemental oxygen during mouth-to-mask ventilation be running at?
10-12lpm
What will adding a reservoir bag to BVM and resuscitator bags flowing at 12-15lpm do to the
oxygen saturation?
increase it from 21% to 90-100%
What is IPPV?
intermittent positive pressure ventilation, "pumping" air or oxygen in through the glottic opening
Will 'pumping' air into the glottic opening guarantee that air will go into the lungs?
No
What is a FROPVD?
Flow restricted oxygen powered ventilation device
Minute volume?
volume of air breathed in and out in one minute (12-15 normally)
Normal ventilation?
movement of air into and out of the lungs is able to maintain good CO2
Hypoventilation?
movement of air in and out of the lungs is unable to maintain the CO2 level below 45mm Hg
What could be a first sign of tension pneumothorax
decreasing compliance
What is the most reliable and effective ventilation technique?
mouth-to-mouth
What can cause or worsen a pneumothorax?
FROPVD
What is BOOTS and what's it for?
Beards, Obesity, Older Patients, Toothlessness, snores/stridor. Predictor of difficult mask
ventilation
What are the five basic pieces of equipment necessary for the initial response to all prehospital
trauma calls?
PPE, backboard with head motion restriction, C-collar, airway kit, trauma box.
What is the mechanism of a tension pneumothorax?
air accumulating in pleural space, increasing pressure on affected side, displacing heart and
trachea, and collapsing the vena cava
What should you be weary of when there is decreased lung compliance in an intubated patient?
Possible development of tension pneumothorax
Who is at risk for development of tension pneumothorax from positive pressure ventilation?
COPD and asthma patients
What are the indications for performing a pleural decompression?
Respiratory distress and cyanosis, loss of radial pulse (shock), decreasing level of consciousness
What is a cardiac tamponade?
rapid collection of blood between the heart and pericardium, causing accumulating blood to
compress the ventricles of the heart preventing ventricles from filling between contractions and
causing cardiac output fail
Signs and symptoms for cardiac tamponade?
Hypotension, narrow pulse pressure, beck's triad
What are beck's triad?
JVD, muffled heart sounds, and paradoxical pulse
What is a paradoxical pulse?
Radial pulse disappears during inspiration
Main differences between cardiac tamponade and tension pneumothorax?
cardiac tamponade pt will be in shock with equal breath sounds and a midline trachea
What diagnoses could result in blunt cardiac injury?
myocardial contusion, dysrhythmia, acute heart failure, valvular injury, cardiac rupture
Most common side injured in heart from myocardial contusion?
Right side, presenting similar to AMI
Signs and symptoms of a myocardial contusion?
chest pain, dysrhythmia, cardiogenic shock, irregular pulse, neck vein elevation,
What percentage of traumatic aortic rupture pt die on scene?
80%
Signs and symptoms of traumatic aortic rupture?
chest or scapular pain, asymmetrical blood pressure in upper extremities or upper extremity
hypertension, widened pulse pressure, diminished lower extremity pulses.
Which side do diaphragmatic tears normally occur and why?
left side because right side is protected by liver
Signs of a diaphragmatic tear?
respiratory distress, diminished breath sounds, bowel sound in chest, abdomen appears scaphoid
if large amount of intestines in chest
What is a pulmonary contusion normally secondary to?
blunt force trauma, penetrating injury, broken ribs, flail segment, etc
Signs of pulmonary contusion?
hypoxemia
Signs and symptoms of traumatic asphyxia?
Cyanosis, swelling in head and neck, tongue and lips swollen, conjunctival hemorrhaging,
pinkness below injury.
What is traumatic asphyxia?
Sudden compression of the heart and mediastinum, transmitting force to capillaries of the head
and neck
Should fluids be given for major traumas to the chest?
Yes, but only enough to keep the systolic at 80-90, with the exception of an aortic tear
What are the different levels of meninges for the brain?
Dura mater (outermost), pia arachnoid (middle, arteries and veins), pia mater (innermost)
Where is cerebrospinal fluid found?
beneath the arachnoid and pia mater
What is a primary brain injury?
immediate damage to the brain tissue that is direct result of on the injury force and is essentially
fixed at the time of injury
What is a secondary brain injury?
result of hypoxia or decreased perfusion of the brain tissue
What is coup and countercoup?
Area of original impact on brain and area on opposite side of original impact
What will an increase to CO2 in the blood do to the brain?
Promote cerebral vasodilation and increases ICP
What will lowering the level of CO2 do to the brain?
cause vasoconstriction and decrease blood flow
What level should CO2 be maintained at in regards to brain injury with ventilations?
35-45, ventilating ever 6-8 seconds
When is ICP considered dangerous?
When is rises above 15mm Hg, herniation occurring at 25mm Hg
What is cushing's reflex?
Our body's response to increased cranial pressure in an attempt to maintain a constant perfusion
pressure.
What are the signs and symptoms of cushing's reflex?
Hypertension, dyspnea, bradycardia
What is CPP?
Cerebral perfusion pressure - the pressure of the blood flowing through the brain
What is MAP?
Mean arterial pressure - the sum of the diastolic BP plus 1/3
When will blood flow cease in the brain?
When the ICP equals the MAP
What is cerebral herniation syndrome?
When ICP forces portions of the brain downward, obstructing flow of CSF and applying great
pressure to the brain stem
Signs and symptoms of cerebral herniation syndrome?
decreasing LOC, coma, dilation of the pupil and outward deviation of the eye on the side of the
injury, paralysis on opposite side of injury, decerebrate posturing, cushing's reflex
What is the only reason you should hyperventilate someone with a TBI?
When they are showing signs of cerebral herniation syndrome
What CO2 level and ventilation ratio should you strive for in a pt with cerebral herniation
syndrome?
30-25mm Hg and 20breaths/min in an adult, 25 in a child and 30 in an infant
What are the definitive signs of cerebral herniation syndrome in the field, allowing you to
hyperventilate a pt?
TBI pt with a GCS of <9 with decerebrate posturing, asymmetrical (or bilateral), dilated, or non-
reactive pupils, or a pt who initially has a GCS of <9 but drops by two points.
What do bilateral dilated and fixed pupils normally represent?
brain stem injury, which is 91% fatal
If the patient has signs of herniation and the signs resolve with hyperventilation, should you
continue hyperventilation?
No
How should an open skull fracture be treated?
adequate oxygen and maintain perfusion, wound dressed without excessive pressure
What is a cerebral contusion and what are the signs of one?
bruised brain tissue, alteration of LOC, weakness, speech problems, personality changes, stroke
symptoms
What is a subarachnoid hemorrhage and what are the signs and symptoms?
intravascular fluid leaking into the brain causing edema - severe headache, coma, vomiting
What is a diffuse axonal injury?
the brain is injured so diffusely that there is a generalized edema, presenting in unconsciousness
with no focal motor defects
What is an anoxic brain injury?
injury to the brain due to lack of oxygen
What is the no-reflow phenomenon
inability of restoring oxygenation and blood pressure to restore perfusion to the cortex after an
anoxic episode of 4-6 minutes or more, with the exception of hypothermic pts.
Signs and symptoms of an acute epidural hematoma?
ICP signs, lapses of consciousness, body paralysis on opposite side, dilated and fixed pupils on
side of injury, rapid death
Signs and symptoms of an acute subdural hematoma?
slow acting ICP, headache, fluctuations in LOC, focal neurological signs (stroke like signs)
What level of the GCS score indicated severe brain injury?
8 or lower
Difference between shock and head injury with ICP?
Shock has narrow pulse pressure, hypotension, tachycardia, tachypnea. ICP has widened pulse
pressure, hypertension, bradycardia, and dyspnea.
What is a primary spinal cord injury?
injury to the spinal cord that occurs at the time of the trauma itself
What is a secondary spinal cord injury?
Injury that occurs from hypotension, generalized hypoxia, injury to blood vessels, swelling,
compression of the cord surrounding hemorrhage, or injury to cord from movement of a
damaged and unstable spinal column
What is neurogenic shock?
shock caused by spinal injury in which the connections of the spine to the adrenal glands are
interrupted and the vasoconstrictors, epi and norepi, are not produced. Without these the blood
vessels dilate and redistribute the blood flow to a larger vascular volume causing a relative
hypovolemia.
What are the signs and symptoms of neurogenic shock?
Hypotension, normal skin color and temp, bradycardia
What should you do before extricating a pt?
Check sensory and motor function in the hands and feet and document, unless it's an emergency
rescue situation situation
What is emergency rescue?
immediate removal without use of SMR in an immediately life-threatening situation
What is rapid extrication?
Rapid removal of a pt in a dangerous position to their health using modified SMR
How should pregnant woman after 20 weeks be transported on a back board?
back board tilted 20-30 degrees to the pt's left side to keep the uterus off the vena cava
How should pt's with airway problems be transported on a backboard?
Tilted to one side to allow gravity to help drain any necessary fluids
How should pt's with unstable pelvic injuries be moved?
Carefully lifted, not rolled, onto a back board or with the use of a scoop stretcher
Should you backboard pt's with penetrating injuries to the trunk?
No, it's a time waster, unless the injures are near the spine.
When is SMR indicated?
Trauma situations where spinal cord injury is potential and where: AMS, evidence of
intoxication, distracting injury, neurological deficit, spinal pain
Which helmet should be removed in the prehospital setting and which should not?
Motorcycle helmets and most other helmets should be removed carefully. Football helmets
should not be removed unless absolutely necessary with conjunction of shoulder pads.
What are the five types of viral hepatitis?
A,B,C,D, and E.
Which hepatitis are healthcare workers most at risk for and why?
Hep B, as it is transmitted through blood (along with sex). Though there is a preventative vaccine
for it.
What does OPIM stand for?
other potentially infectious material, such as synovial fluid, saliva, CSF, etc
What's the likelihood of becoming infected by HCV after a high-risk needle-stick?
1.5%
Is there a vaccine for HCV?
No
What are the two main differences in how HBV and HIV are spread?
HIV does not survive outside the body and HIV is transmitted far less efficiently than HBV
In general, which is more significant in transmission of diseases? Large bore needles or scalpels?
Large bore needles
What is post-exposure prophylaxis?
the administration of medications (within hours) wto prevent infection from the agent which the
person was exposed.
What are some risk factors for tuberculosis infection?
homeless people, certain immigrant population, patients at risk for HIV infection, people who
live in congregate settings
Signs and symptoms of tuberculosis?
Productive cough that lasts longer than three weeks in conjunction with two of the following:
bloody sputum, weakness/fatigue, weight loss, fever/chills, night sweats, hoarseness
Is tuberculosis a virus or a bacteria?
Bacteria - mycobacterium tuberculosis
How should healthcare providers prevent infections from tuberculosis to themselves?
Placing a mask on the pt.
What actions should healthcare providers take to prevent MRSA and similar diseases?
Gloves, good hand-washing, and good cleaning of equipment, including exercise equipment
What are the conditions in which resuscitation should NOT be performed in traumatic
cardiopulmonary arrest?
Blunt trauma with no breathing, pulse, or organized rhythm on ECS on arrival, penetrating
trauma with no breathing, pulse, pupillary reflex, spontaneous movement, or organized ECG
activity, trauma with injuries obvious signs incompatible with life, and trauma with evidence of
significant time lapse since pulselessness
What is the most common cause of traumatic cardiopulmonary arrest?
Hypoxemia
What is ARDS?
Adult Respiratory Distress Syndrome,
What will PPV do to a person in shock?
Lower their blood pressure, as PPV diminishes venous return to the heart, lowering cardiac
output
What is a common cause of myocardial contusion?
Deceleration accidents
What is Commotio cordis?
V-fib triggered by blow to the anterior chest wall during cardiac repolarization
What does traumatic cardiopulmonary arrest usually present as from an electrical shock?
V-fib
Do patients who are in TCPA after shock have a good chance of survival with resuscitation
efforts?
Yes they do, as most of the time they are experiencing arrest due to the electricity and not the
trauma before or after the shock.
Why is the resuscitation rate for TCPA patients so poor?
The fact that many of them have been hypoxic for a prolonged period of time before the arrest
occurred, which causes severe acidosis
If a pt is found in asystole after massive blunt trauma, should they be resuscitated?
No, their resuscitation can be terminated in the field
When should you discontinue chest compressions?
There is a palpable pulse or resuscitation is terminated
If a pt is in TCPA, how many liters of saline should be given?
2 liters rapidly
If a pt is suffering from a massive hemothorax, how many liters of saline should be given?
2 liters rapidly
What rate is a good ventilation rate for pt in TCPA?
1 breath every 8 seconds
What problems are TCPA patients normally suffering from?
Breathing problem or circulatory problem
What drug class are constricted pupils an indication of?
Opiates, and early use of barbituates
Which drug classes are dilated pupils an indication of?
amphetamines, cocaine, hallucinogens, and marijuana, late dose of barbituates
What is interactive style?
a positive, non-judgemental interactive style through speech and body language
In impaired pt's, what kind of questions should you ask and why?
Close-ended questions, due to their inability to concentrate
How should you treat someone who is impaired due to alcohol?
IV thiamine and glucose, D50W if indicated, and watch for hypothermia
How should you treat someone who is impaired due to amphetamines?
Monitor for seizures and dysrhythmias, treat seizures with diazepam or lorazepam
How should you treat someone who is impaired due to cocaine?
Monitor for seizures and dysrhythmia, treat rhythm disorders, avoiding beta blockers, as they
increase myocardial ischemia
How should you treat someone who is impaired due to hallucinogens or marijuana?
Provide reassurance
How should you treat someone who is impaired due to narcotics/opiates?
Try naloxone, watch for hypothermia, hypotension, and respiratory depression
How should you treat someone who is impaired due to sedatives?
Try naloxone, and consider flumazenil, watch for hypothermia, hypotension, and respiratory
depression
Normal perfusion of body tissues requires these four intact components:
Intact blood vessels, oxygenation, adequate volume of fluid in the system: RBC's and plasma,
and a functioning pump!.. the heart
What is the equation for blood pressure?
BP=COxPVR
What is the equation for cardiac output?
CO=HRxSV
What term describes a condition that occurs when the perfusion of the body's tissues with
oxygen, electrolytes, glucose, and fluid becomes inadequate to meet the body's needs?
SHOCK = LOW TISSUE PERFUSION
What reaction does inadequate oxygen perfusion cause the body to respond with to increase both
HR and strength of heart contractions and constrict peripheral blood vessels?
The sympathetic nervous system is stimulated increasing release of epi and norepi
Blood pressure should be monitored frequently to determine adequate perfusion but a the
presentation of hypotension is a _____ sign of poor perfusion.
Late
Compensated shock consists of the following signs and symptoms:
Weakness and lightheadedness, Thirst, Pallor, Tachycardia, Diaphoresis, Tachypnea, Decreased
urinary output, and weakened peripheral pulses (thready)
Decompensated shock consists of the following signs and symptoms:
Hypotension, AMS, and cardiac arrest
What is a normal pulse pressure and what presents as an initial sign of shock within BP pulse
pressure?
120/80 = pulse pressure of 40mm Hg normally and narrowing of this pressure initially presents
as the diastolic rises toward systolic pressure when shunting of extremities begins (resulting in
the pallor seen in shock)
At which point does shock move from compensated to decompensated?
As BP begins to drop this signifies the "crash" The acidosis causes a lack of catecholamines
worsening the drop in BP
What percent of blood volume is lost in Early Shock? (compensating)
15% - 25%
What percent of blood volume is lost in Late Shock? (decompensating)
30% - 45%
Cap refill time should be approx.?
2 secs. (High correlation during late shock but of relatively little value for detecting early shock)
Cap refill useful in small children in which a BP may be hard to obtain
What is one of the first/earliest signs of illness?
Tachycardia
Elevated pulse rates (generally above 100) are never normal!
Elevated pulse rate is one of the first signs of shock. Over 100 should suspect hemorrhage until
proven otherwise
During ITLS primary survey, what pulse rate should be a red flag for shock?
120 or above
Patients with traumatic Hypotension may develop..
Relative bradycardia
What percent of patients with bleeding in the abdomen won't show tachycardia?
20%
What medications would affect the intrinsic heart rate and mask symptoms blocking tachycardia
even with excessive blood loss?
Beta-blockers and calcium channel blockers
Are children able to increase their stroke volume?
No, the cardiac output is dependent on heart rate alone
Children in decompensated shock may develop..
Bradycardia
Falling level of CO2 indicates what about the patients?
Hyperventilation and inadequate oxygen
Worsening shock is indicated by EtCO2 less than..
20's or below
What are the three major classifications of shock?
Low-volume shock (absolute hypovolemia) = major fluid loss
High-space shock (relative hypovolemia) = spinal injury; vasovagal; sepsis etc., to dilate vessels
Mechanical shock (cardiogenic or obstructive) = pump problem; myocardial contusion or
infarction; or pericardial tamponade, tension pneumothorax, or massive PE.
Hypovolemic patients usually present with..
Pale, tachycardia, and FLAT neck veins
The average size adult has about 5L of blood within the vessels but how much blood can the
vascular system hold?
25 Liters of blood if the arterioles were going to fully dilate
What is the clinical presentation of neurogenic shock vs hemorrhagic shock?
No catecholemines released in neurogenic shock so no pallor, tachycardia, or sweating will
present. (the pt will have a decreased BP but the HR will be normal or slow and the skin usually
p/w/d.)
How many liters of blood can the heart pump in one minute?
5 liters
What is the time frame from development of traumatic cardiac tamponade to circulatory arrest?
5-10 minutes
General management of Post-traumatic shock states
Control bleeding, administer high-flow oxygen, load and go
How do you manage shock with bleeding controlled?
Put the pt in a horizontal position, administer high flow oxygen, transport immediately, obtain
large bore IV access enroute, give normal saline at a rate of 20ml/kg, place the pt on ECG
monitor, apply pulse ox and capno, perform ongoing exam.
How do you manage external Hemorrhage that you cannot control?
Apply direct pressure, put the pt in a horizontal position, don't hesitate to apply a tourniquet,
apply hemostatic agent (quik-clot), administer high flow oxygen, transport immediately, gain IV
access, give only enough saline to maintain systolic BP of 80-90mm hg, and continue to monitor.
How do you manage shock due to internal hemorrhage?
Transport immediately, place pt in horizontal position, administer high-flow O2, gain IV access
with large bore IV caths, administer normal saline enough to maintain peripheral perfusion,
monitor the heart, perform ongoing exam.
How do you manage a pt's blood pressure with a pt with severe head injury and a GCS of less
than 8?
Keep their BP at 120 to keep their cerebral perfusion at 60
What changes occur in elderly in regards to their airway?
tooth decay, gum disease, dental proethetics
What changes occur in elderly in regards to their respiratory system?
Circulation to the pulmonary system decreases 30%, reducing the CO2 and O2 exchange,
decrease in chest wall movement, rapid breathing, breathing capacity decrease
What changes occur in elderly in regards to their cardiovascular system?
CO and SV decrease, higher BP
What changes occur in the elderly in regards to their neurological and sensory function?
brain shrinks, space creation between brain and skull, hardening and narrowing, and loss of
elasticity of some arteries in the brain, decreased blood flow to brain, slowing of sensory
perceptions, higher pain tolerance, decreased ability to compensate for shock
What changes occur in the elderly in regards to their thermoregulation?
They can't maintain body temperature as easily
What changes occur in the elderly in regards to their renal system?
decrease in function nephrons
What changes occur in the elderly in regards to their musculoskeletal system?
changes in posture, osteoporosis, diminished subcutaneous tissue, weakening of muscles
What changes occur in the elderly in regards to their GI system?
Decrease saliva, motility, gastric secretion, decreased ability to metabolize medication
What changes occur in the elderly in regards to their immune system?
It decreases
What changes may increase susceptibility to injury in the elderly?
slower reflexes, failing sights, hearing loss, arthritis, fragile skin and blood vessels, fragile bone
What's important to note about assessing elderly patients?
Elderly patients may have chronic diseases and may not have the same sensory reactions as
younger people, and also may have multiple injuries.
What is the intrathoracic abdomen?
The par of the abdomen that is enclosed by the lower ribs, contains the liver, gallbladder, spleen,
stomach, and transverse colon
What is the true abdomen?
that part of the abdomen from the lower ribs and including the pelvis but anterior to the
retroperitoneum, contains the large and small intestines, a portion of the liver, and the bladder,
ovaries, uterus, and fallopian tube
What is the retroperitoneal abdomen?
the part of the abdomen behind the thoracic and true portions of the abdomen separated from the
other abdominal regions by the thin retroperitoneal membranes, includes kidneys, ureters,
pancreas, posterior duodenum, ascending and descending colon, abdominal aorta, and the
inferior vena cava
Can hemorrhage severe enough to cause shock occurring in the retroperitoneal space show
obvious signs of abdominal distension?
No, it doesn't necessarily have to show distension
Where are the iliac blood vessels located?
in the pelvic portion of the retroperitoneal space (major blood vessels)
What mortality rate does blunt trauma to the abdomen have?
10-30%, due to frequency of accompanying injuries to other systems
Will pt's with blunt trauma to the abdomen always be in pain?
No, in fact they may show little signs of injury as well
Can severe pain from rib fractures be a distracting injury to less noticeable abdominal pain?
Yes
What is blast effort?
gunshot wounds include trauma through penetration, its fragments, or the energy transmitted
from the bullet's mass and velocity
What mortality rates do patients with penetrating trauma to the abdomen have?
5-15% because of greater incidence of injury to abdominal viscera
What is the mortality rate from abdominal stab wounds?
1-2%
What is associated with up to 50% incidence of significant intra-abdominal injuries?
penetrating trauma in the gluteal area
What is the major cause of preventable mortality in abdominal trauma?
Delayed diagnosis and treatment, so don't worry about extensive history
If tenderness or guarding is present in the prehospital setting, there is usually significant blood in
the abdomen causing irritation of what?
the peritoneum, which is an indication of imminent shock
When should the genitalia be visualized for bruising or hemorrhage?
The Secondary survey
Should you probe abdominal wounds with your finger or an instrument?
No
Does auscultation of the abdomen in the field usually offer further useful information?
No
What is CABC?
Control external hemorrhaging, Airway, breathing, circulation
In abdominal trauma, how should systolic pressure be maintained?
kept at 80-90mm Hg via IV fluids
If you have a long transport and evisceration, how can you manage the pt condition?
Apply non-adherent material, such as plastic wrap or aluminum foil, to prevent drying of the
gauze and intestines
What changes occur during pregnancy?
increase blood volume, increase cardiac output, decrease blood pressure, decrease volume in
thoracic cavity , increase in both red blood cells and plasma, causing pt to appear anemic, gastric
motility decrease (assume pt may vomit)
In the first trimester (1-12 weeks), what's the viability, cause of vaginal bleeding, fetal heart
tones, and height of fundus above pubis symphysis?
Fetus not viable, there is potential for miscarriage, heart tones not obtainable, and it's difficult to
measure
In the second trimester (13-24 weeks), what's the viability, cause of vaginal bleeding, fetal heart
tones, and height of fundus above pubis symphysis?
Potential viability, potential miscarriage, 120-170 bpm, and it's halfway to umbilicus equal 16
weeks, and to the umbilicus equals 20 weeks
In the third trimester (25-40 weeks), what's the viability, cause of vaginal bleeding, fetal heart
tones, and height of fundus above pubis symphysis?
Fetus viable, potential birth problem, 120-160 bpm, and 1 cm equals 1 week until 37 weeks, then
uterus height decreases as the baby settles into the pelvis.
What effect does hypovolemia shock and vasoconstriction have on the uterus?
leads to reduction in uterine blood flow by 20-30%, causing the pregnant patient to lose up to
1500cc of blood before any detectable change is noted in her blood pressure
What fetal mortality rate is associated with a mother in shock, and why?
80%, since the baby does not get adequate oxygen due to the lateness of acknowledgement of
blood pressure dropping, therefore always give 100% oxygen to pregnant mother by NRB
What is supine hypotension syndrome?
hypotension in mother who is greater than 20 weeks in the supine position due to weight of
uterus pressing on inferior vena cava and decreasing venous return
How should you manage supine hypotension syndrome?
Tilt or rotate backboard 15-30 degrees to the patient's left, elevate the right hip 4-6 inches with a
towel and manually displace the uterus to the left
What is the most common cause of fetal death in trauma?
Maternal death
What is most often seen in pregnant patients who have been in MVC's?
fetal distress, fetal death, abruptio placenta, uterine rupture, preterm labor
What is the most common cause of death in pregnant patient involved in MVCs?
head injury
How many pregnant women experience domestic abuse?
1 in 10
What kind of signs and symptoms would you see in an abused pregnant woman?
proximal and midline injuries, face and neck, low birth weight
Is fluid requirement for a pregnant patient greater than a non-pregnant woman?
Yes, so especially in burns, fluid replenishment is very important
Whats the ventilation rate for a child less than 1 year of age?
20/minute
What is the ventilation rate for a child greater than 1 year?
15/minute
What is the ventilation rate for an adolescent?
10/minute
How can you choose sizes of intubation tubes for a child?
Broslow tape, choose a tube that is the diameter of the child'd tip of finger, or use the equation:
4+(age/4)=size
When should you stop trying to intubate a child?
When their O2 drops below 90, or 15 seconds have passed
How much blood volume does a child have?
80-90mL/kg
How should you control bleeding in a child?
Using a gloved hand and 4x4 sterile gauze and firm pressure until the bleeding is controlled
GCS for eyes?
4-spontaneously, 3 - verbal command, 2 - to pain, 1 -no response
GCS for best motor response?
6 - obeys, 5 - localizes pain, 4 - flexion, withdrawal, 3 - flexion, decorticate, 2 - extension,
decerebrate, 1 - no response
GCS for Best verbal response?
5 - oriented and converses, 4 - disoriented and converses, 3 - inappropriate words, 2 -
incomprehensible sounds, 1 - no response
How should you treat shock with fluids in a child?
Fluid resuscitation - initial bolus of 20mL/kg, if no response, another 20mL/kg. If you can't do an
IV in 90 seconds, IO the child.
What's the most common cause of death in pediatric patients?
Head trauma
Are the liver and spleen larger or smaller in pediatric pts compared to adults?
Larger
Where do veins, nerves and arteries run in regards to the extremities?
They travel near the bone, across the flexor side of joints or very near the skin
How much blood can be lost in a closed fracture?
2 liters
How much blood can be lost in a pelvic fracture?
1 liter as typically a fractured pelvis is broken in two places
How should dislocations and fractures be managed?
Typically, they should be splinted in the position in which they are found with adequate padding
and stabilization, unless there's a long transport, you should pull gentle traction to restore distal
pulses
Should hemostatic agents be used in open abdominal or chest wounds?
No
When is the only time you should change from ABC to CABC?
Life-threatening hemorrhage
How should you dress an open wound where bleeding is controlled?
Moist sterile dressing and bandage. Leaves or gravel should be removed if possible, smaller
pieces of contamination can be irrigated
How should an amputation be managed with relatively controlled bleeding?
cover with damp sterile dressing and an elastic wrap that will apply uniform, reasonable pressure
across the entire stump, if bleeding uncontrolled, use tournequet (bring amputated parts with)
How should you transport and amputated part?
rinsed off, wrapped in sterile gauze and placed in a plastic bag (don't use dry ice. never place
directly on ice)
What can cause of a loss of sensation in an extremity?
disruption, swelling, or compression by bone fragments or hematomas
What is a sprain?
stretching or tearing of ligaments of a joint because of a sudden twist
What is a strain?
stretching or tearing of muscle of musculotendinous unit
When should you remove an impaled object?
When it's in the back, in the middle of the chest and you must do CPR, or when it's in the neck or
anywhere that is obstructing the airway
What is compartment syndrome?
a condition in which increased tissue pressure in a confined space causing decreased blood flow,
leading to hypoxia and possible muscle, nerve, and vessel impairment
What are the signs and symptoms of compartment syndrome?
pain, pallor, pulselessness, paresthesia, and paralysis
What toxins are released in a crushing injury that can cause further damage?
myoglobin, potassium, phosphorous, lactic acid, and uric acid, causing cardiac dysrhythmia
and/or kidney damage (Crush/compression syndrome)
What associated injuries occur with foot injuries?
Possible lumbar injuries
What associated injuries occur with injury to the knee when the patient is in a sitting position?
possible hip injuries
How long should a tourniquet be on for?
2 hours or less
Should you cover a tourniquet?
Never
How long should you hold pressure for on hemorrhage?
at least 2 minutes or until bleeding is controlled
How can internal bleeding from unstable pelvic fracture be decreased?
Circumferential stabilization of the pelvis
How much blood volume can be lost in a bilateral femur fracture?
Up to 50%
Where are hip fractures most commonly fractured at?
The narrow neck of the femur
How does posterior hip dislocation present and how should it be managed?
inward rotation of the leg, pillow and splint, no traction or straightening
How does the anterior hip dislocation present?
Much like a fractured femur, pillow and splint, unlikely ability to bring the leg forward in line
with the body, don't try as clot can form and be moved once hip is reduced
How should knee dislocation be managed?
If distal pulses no present, apply gentle traction by hand. Splint in position if resistance is met
How should patellar dislocation be manage?
Splinted with pillow under knee, Straightening leg if possible, which might spontaneously reduce
this injury
What is the most frequently fractured bone in the body?
The clavicle
Injury to which nerve would cause an inability of the patient to lift the hand?
radial
Should an attempt be made to straighten or apply traction to an injury to the elbow?
Never
How should the toxins of crush syndrome be addressed?
Since the blood becomes acidotic, large volumes of fluid and alkalizing agents such as sodium
bicarb and osmotic diuretic like mannitol should be administered
If administration of fluids or medications are not possible prior to releasing the entrapped body
area, what should you consider?
Application of a tourniquet to reduce systemic release of toxins in crush syndrome
Why are children at greater risk for severe injury following a burn?
Their skin is thinner, making hypothermia more likely to occur
Prior to secondary transport, what should have been completed by the physician?
stabilization of respiratory and hemodynamic function, assessment and management of
associated injuries, review of lab data, insertion of nasogastric tube, foley, assessment of
peripheral circulation, proper arrangements with receiving hospital
What is the parkland formula?
4mL x %burn area x body weight (kg) = amount of ringer's lactate or normal saline needed in
first 24 hours
How much fluid should be given the first 8 hours of a burn pt?
half of the fluid calculated in the parkland formula, the other half in the next 16 hours
What is a radiation injury?
injury to the skin and tissues from the effect of ionizing radiation. The injury is caused by
radiation breaking molecular bonds within the cell and can't be differentiated from thermal burns
by appearance alone
How quickly to radiation burns develop?
Slowly, over the course of days
Are pt's with radiation burns, radioactive?
Not unless they are contaminated with radioactive material, which we are unable to
decontaminate them in field
What is the largest organ of the body?
The skin, which is made up of two layers, the epidermis and the dermis
Where are sensory nerve, hair follicles, sweat glands, and oil glands located in the skin?
The dermis
What are the types of burn injuries?
Thermal, electrical, chemical, and radiation
What is the portion of the skin that are necrosed by the thermal insult referred to as?
Zone of coagulation, which is irreversible
What is the area around the zone of coagulation called?
Zone of stasis, blood flow is compromised but tissue may be restored
What is the are around the zone of stasis called?
Zone of hypermia, where there is increased blood flow to the tissues as a result of inflammatory
mediators released by damaged skin
What is eschar?
skin protein becoming denatured and hard, forming a firm leatherlike covering
1-2 days after a burn, what can occur to the tissue?
Further damage, increasing the burn depth due to decreased circulation to the area
Do patients who have been severely burned usually expire in the initial postburn period?
Not normally from the burn injury but typically from associated injuries
How should the airway be managed in a burn pt?
NRB and intubation if needed due to developing inhalation injuries
How should you manage a burn?
room temp irrigation for no more than 2 minutes, dry sheets, keep pt warm
Should you try to pull of clothing adhered to a burn pt?
No, just cut around it
Should you start IV on a critically burned pt?
Not if it delays initial transport
Should you give pain medications to burn pts?
Only after consulting a medical director
What is a flash burn?
burn that occurs when there is some type of explosion, but no sustained fire, normally results in
1st or 2nd degree burns
What is the most common cause of early death associated with a burn injury?
CO poisoning and asphyxiation
What is the mechanism of death in CO poisoning?
cerebral or myocardial ischemia or MI due to progressive cardiac hypoxia
How should CO poisoned pts be treated
High flow 100% oxygen, as the effects can be reduced with oxygen, taking up to 7 hours
Where are heat-inhalation injuries confined to?
Upper airway, with the exception of steam burns or inhalation of a flammable gas which ignites
causing thermal injury to the alveoli
What's an indication for swelling of airway in a burn?
Stridor, hoarseness
Can aggressive fluid resuscitation harm a burn victim?
Yes, in an inhalation injury as it increases swelling in the airway
How should you intubate a pt with inhalation injury?
Nosotracheal intubation is preferred or by paralysis and drug assisted intubation
What medication should be used to treat bronchospasm in a burn pt?
inhaled beta agonist
What's a side effect of chemical burn injuries?
internal organ failure due to absorption of chemicals
What are the steps of removing the source of the chemical burn?
PPE, remove clothing (place in plastic bag), flush chemical with clean water (brush off large
debris first), remove agents adhering to skin by wiping or scraping
What are the factors that determine severity of electrical injury?
Type and amount of current, path of current through body, duration of contact with current
source
What is the most serious and immediate injury resulting from electrical contact?
cardiac dysrhythmia, particularly ventricular dysrhythmia
What can be present in an electrical injury other than burns and cardiac dysrhythmias?
fractures and dislocations due to violent muscle contraction
What is rhabdomyolysis?
disintegration or dissolution of muscle. Releasing large amounts of myoglobin into the blood,
which can precipitate in the kidneys, causing renal failure. A result of burns
What is the most serious effect of a lightening strike?
Cardiorespiratory arrest, due to massive current acting like a defibrillator to briefly stop the
heart, which is why CPR should be immediately initiated as there is a higher risk of resuscitation

Abnormal adult respiratory rate that needs intervention (airway, bag assist etc..)
<8 and >24
When to do spinal mobilization in initial assessment
Right after airway
A,(C-spine)B,C,(Control bleeding)
3 things that stop your assessment right away.
scene hazard, airway obstruction, cardiac arrest
During the Primary Survey, when do you Control major bleeding
In the Initial assessment
Airway, breathing, circulation, control bleeding
When the injury occurs, what clock just started?
The "Golden Period" (used to be the golden hour)
Max length of your initial assessment
2 min.
Max length of "load and go" scene time?
Critical trauma scene time is 5 min. or less.
ELM stands for?
External Laryngeal Manipulation
Selick vs. ELM
Selick is when you close the esophagus to prevent air in the gastric.
ELM is a manipulation technique to move the airway into alignment for intubation.
SMR stands for?
Spinal Mobilization RESTRICTION
Best GCS score is?
Eyes 4
Verbal 5
Motor 6
Total 15
1st action of initial assessment?
Scene safety
Who is responsible for assessments of the patient?
Team leader only
Focused or Rapid trauma survey for a generalized, significant MOI?
Rapid trauma assessment
How much does the tube move with neck flexion or extension?
2-2.5cm
What team member can determine level of consciousness?
team leader only
Most important thing to do when a child is unresponsive with obvious respiratory
disress, tachypneic and shallow, weak slow pulse?
assist ventilations
What angle and position do you tip or roll a pregnant woman?
Left lateral side, rolled up 15-30 degrees
Adequate respiratory rate for an adult?
10-12 per minute
every 5-6 seconds
What does stridor indicate in a trauma patient?
Airway obstruction
Major pediatric sign of shock?
tachycardia
ETCO2 level for impending respiratory collapse?
<30
BOOTS acronym
Difficult BVM items
B-Beards
O-Obesity
O-Older
T-Toothless
S-Snoring
In a head injury what do you maintain BP at?
110-120 mmHg
What is a red flag vital sign for shock?
Tachycardia BP >120
What do you suspect in trauma with a BP >100?
Hemorrhage
Minimum BP for perfusion?
50 to 60 mmHg
Max length of time to have a tourniquet on?
2 hours
List 3 shock syndromes
Low Space shock
High space shock
Mechanical shock
Absolute hypovolemia
Low Space shock
Relative hypovolemia
High Space shock
Cardiogenic or Obstructive shock
Mechanical shock
Low Volume shock
(Absolute hypovolemia)
Hemorrhage, vomiting, "third spacing" from burns, peritonitis
High Space shock
(Relative hypovolemia)
Spinal injury, Vasovagal syncope, Sepsis, drugs that dilate
Mechanical shock
(Cardiogenic or Obstructive shock)
Pump problems, Myocardial contusion, Infarction, pericardial tamponade, tension
pnumothorax, massive pulmonary embolism
Breaths per min. with severe head injury
<8
3 places you do you NOT use homeostatic agents?
head
abdomen
chest
4 things to check with an altered patient
Pulse
Pupils
LOC
Sugar
What 2 items would determine how much fluids to give IV for shock?
Enough to have peripheral pulses.
SBP 80-90 mmHg
Normal ETCO2
35-45 mmHg
Parkland formula
Amount of fluid required in 24 hours (ml) = 4 Patient's weight (kg) Percent body
surface area (BSA) involved in burns
What is the difference between simple pneumo and tension pneumo
SHOCK
When is the only time you hyperventilate a patient?
Suspected herniation
What ETCO2 do you maintain during hyperventilate?
30-35 mmHg
Important procedure when dealing with a lightening victim?
Monitor for dysrthmias
Drug to give for crush injury?
Sodium bicarbonate
Significant MOI that indicate "load and go"?
Bilateral femur fractures
shock
pericardial tamponade,
altered level of conscious
abnormal breathing
uncontrolled bleeding
distended abdomen
Penetrating wound to torso
Indications for hyperventilation
Cushing's triad, posturing, sudden decrease in GCS
Normal Fluid replacement
20ml/kg
What to do if Mom is nonsalvagable (death during CPR)?
continue cpr to preserve the neonate
Significant changes in pregnant women occurs with how much blood loss?
30-35%
Two things to do with ped with a head injury?
Oxygenate
Ventilate
BP when shock is at its worst?
<50 mmHg
Minimum Cerebral Perfusion Pressure (CPP)
50 mmHg
Hyperventilation rates for adult, child and infant?
adult 20
child 25
infant 30
Preferred airway technique for pediatric?
BVM
Endotrachial tube is elective
Best way to determine pediatric ET size and other interventions?
Always use Braslow tape
Pediatric blood volume
80-90 mL/kg
Pediatric fluid replacement
20 mL/kg (same as adult)
GCS for severe head injury
<8
Significant indication of ICP
Irregular or elevated breathing
Minimum child BP
<80 mmHg
Minimum young adult BP
<70 mmHg
Treatment for flail chest
bulky dressings
Key HemoPnumo symptom
Flat neck vein
Key Pulmonary Contusion symptom
Hypoxemia
Lack of JVD, flat neck vein
Difference between simple and tension pnumothorax
SHOCK!
Unstable pelvis
break in 2 or more places
Amount of blood loss per femur fracture
1L
Pt is in a house fire and comes out altered. What is most likely?
Smoke Inhalation (test answer)
Key thing to do for pregnant moms?
High flow O2 because moms body will compensate for mom and not the baby.
Vital sign differences with pregnant moms
Blood volume increased 45%
Increased HR of 10-15 bpm
BP lower by 10-15mmHg
Don't work a traumatic arrest except with:
Peds
Hypothermia
Lightnening
Key Cardiogenic shock symptom
Lower HR
Normal skin signs and temp
Location of anterior needle decompression
Directly over the top of the 3rd rib, 2nd intercostal
Question with a bent steering wheel and NO other major S/S.
Myocardial contusion
Question an unresponsive victim in a pond.
No "C" spine is the answer
ITLS Primary Survey items
Scene Size-up
Initial Assessment
MOI
Rapid Trauma Survey OR Focused Exam
Load & Go situation?
Needle decompression lateral site?
Anterior axillary, intersection of nipple (fourth rib) and anterior axillary line (pg134)

Insert in 3rd or 4th intercostal, 90 degrees


Herniation Syndrome Symptoms
(Swelling of the brain causing coma. End result is squirting brain through the foramen
magnum)

Dilated pupils
Paralyis on one side
High BP, Bradycardia -Cushing's reflex
Decerebrate posturing
GCS drops from <9 to <2
Downward deviation of the eye on injury side
Herniation Syndrome field treatment
Hyperventilate adult to 20 bpm or every 3 sec.
Keep ETCO2 at 30-35 mm Hg
Chest Decompression Indications
Respiratory Distress And cyanosis (rapid and shallow)
Loss of radial pulse (weak and thready)
Skin diaphoretic
Decreasing LOC

Possible:
Tracheal deviation
Neck vein distention
Hyperresonance
Early signs of hemorrhagic shock
Narrowing pulse pressures
(vasoconstriction increases diastolic pressure)
Falling height of capnography wave form
Neurogenic shock signs
Decreased BP
HR normal or slow
Skin is warm, dry and pink
Spinal cord injury
Among the following, what is the most common cause of preventable trauma death in
the injured adult patient?
Hemorrhagic shock
Which of the following is most typical of early, neurogenic shock?
Decreased pulse, warm and dry skin
A 54 year old man is involved in a motor vehicle collision. The steering wheel is bent.
During your initial assessment you note his skin is pale and his radial pulses are
present. Breath sounds are clear, heart tones are not muffled. Which one of the
following is most consistent with these?
Cardiac contusion
What is the most common cause of cardiopulmonary arrest in the trauma patient?
Hypoxemia
Which of the following regarding flow restricted oxygen-powered demand valves is
TRUE?
Use of them is controversial and they may not be recommended for use
Changes in which of the following is most useful to follow in the child with head injury?
Level of consciousness
An unrestrained 18 year old male on the way to a post-graduation party leaves the road,
bounces through a ditch and hits a tree. You find him behind the bent steering wheel,
unconscious, cool, pale and clammy, with labored respirations of 30 and shallow,
thready radial pulses of about 120, flat neck veins, trachea midline, an asymmetrical
chest with absent breath sounds on the left. You assume he has a:
Massive hemothorax
A 25 year old woman is involved in a motor vehicle collision. The following findings are
noted on primary survey: rapid breathing, cyanosis, weak and rapid pulse, delayed
capillary refill, distended neck veins, midline trachea, contusion to anterior chest and
equal breath sounds. The most appropriate action for you to take is:
Immobilize the cervical spine, place the patient on backboard and transport
in which of the following situations should an emergency rescue be used?
presence of toxic fumes
a 34 y/o man has a gunshot wound to the right groin area. Arterial bleeding, which
cannot be controlled with direct pressure, is coming from the wound. The patient
appears confused, diapheuretic and has weak peripheral pulses. what is the appropriate
fluid resuscitation for this patient?
apply a hemostatic agent and gain intravenous access given enough fluid to maintain
peripheral pulses

You might also like